Crim Law/ Crim Pro Flashcards
ALA Quizzes, Kaplan Quizzes, Kaplan Qbank, MAYBE/MOST LIKELY CLS Quizzes
A man decides to steal a new car. He walks to a car dealership and tells the salesman that he wishes to test drive a car. The salesman hands the man the keys. The men stars the car, drives off and never returns the car.
The man is guilty of:
(A) Larceny
(B) Larceny by trick
(C) Embezzlement
(D) Larceny by false pretenses
(B) Larceny by trick
A defendant wished to see his high school basketball team win the state championship. During an important game, the defendant pulled out a gun and shot at the leg of a key player on the opposing team. The defendant intended only to inflict a slight wound so that the opposing player would be unable to complete the game. When the defendant fired the shot, he unintentionally hit a player on his own high school team in the chest, killing him instantly.
What is the most serious crime that the defendant can be convicted of?
a. Murder.
b. Voluntary manslaughter.
c. Involuntary manslaughter.
d. battery.
a. Murder.
A husband and wife arrived at a hotel and gave their luggage to a bellhop to bring up to their room. The husband then went to make a phone call. When the bellhop arrived at the room, the wife seduced him and he did not resist her advances. The husband got back to the room to find the bellhop and his wife in bed together. Filled with rage, the husband grabbed a lamp and smashed it over the bellhop’s head, killing him instantly. The husband intended to hurt the man, but not to kill him. Is the husband guilty of voluntary manslaughter?
Yes, because the husband was adequately provoked.
A shopper enters a convenience store and picks up a gallon of milk. He approaches the counter and hands the clerk a $5 bill and leaves the store. The clerk places the bill in the cash register. At the end of his shift, the clerk opens the cash register, takes a $5 bill, and leaves for home.
The clerk is guilty of:
(A) Larceny
(B) False pretenses
(C) Embezzlement
(D) Conversion
(A) Larceny
A driver was pulled over by a motorcycle officer for speeding. The officer approached the driver’s window and asked the driver to turn off her car and present her license and registration. While she fumbled through her purse, he noticed an illegally modified gun
sticking out from under the passenger’s seat. He asked the driver to get out of the car, he patted her down and then he searched her purse and wallet, which the driver set on the front seat of the car when she got out. The officer found a small bag of cocaine tucked in the folds of her wallet. The driver was arrested.
Is the bag of cocaine admissible evidence?
(A) Yes, because the search was conducted pursuant to a lawful traffic stop.
(B) Yes, because seeing the illegally modified gun in the car gave the officer probable cause to believe that the driver had additional illegal weapons.
(C) No, because the cocaine was in the driver’s wallet.
(D) No, because the officer was only permitted to pat the driver down to feel for weapons.
(C) No, because the cocaine was in the driver’s wallet.
A novice hunter was out with a friend on a hunting trip. The hunter had brought a new gun along and wanted to try it out, so he waited until the two had walked into deep brush and then he fired several shots off. One of the shots ricocheted off a tree and struck the friend, killing him.
If the hunter is charged with involuntary manslaughter, what would be his best defense?
A The hunter had never tested the gun before.
B This was the friend’s first hunting trip.
C The hunter could not see the friend when he fired the gun.
D The hunter had never seen a bullet ricochet off the trees before.
The correct answer is: The hunter had never seen a bullet ricochet off the trees before.
Discussion of correct answer: Involuntary manslaughter is an unintentional killing where the defendant was either reckless or criminally negligent in his action, causing another’s death. Here, if the hunter had never seen a bullet ricochet of a tree before, that fact supports a finding that he was neither reckless or criminally negligent. Having never seen it happen, the hunter would be less likely to have known or been certain that firing the gun would cause a bullet to ricochet. Such a fact negates the mens rea for involuntary manslaughter and would supply the hunter with his best defense. As a test tip, involuntary manslaughter is an unintentional crime where the defendant is either reckless or criminally negligent in his actions, causing another’s death. It is helpful to think of a known example of a crime to see if the facts of the example are similar the facts of the question. A known example of involuntary manslaughter is when the defendant is driving a car while intoxicated and has an accident where someone dies. Drinking and driving is clearly unreasonable behavior. In this question, answer choice D could help the defendant, because if he had never seen a bullet ricochet off trees before, he may have been acting reasonably by firing his gun.
After being laid off from his job, a bus driver was hard up for cash. He borrowed $3,500 from his friend, a limo driver, promising to repay her the money within three months. Six months elapsed without the bus driver repaying the limo driver. The limo driver has left
messages for the bus driver, but he has not returned any of her calls. Finally, the limo driver decides that the most effective approach is probably to speak with the bus driver in person. She asks her friend to accompany her for moral support. The friend
misunderstands, however, and believes that the limo driver wants him to use physical force to “persuade” the bus driver to pay the limo driver the money he owes. When the limo driver and the security guard arrive at the bus driver’s apartment, the bus driver opens the front door. Before the bus driver recognizes who is at his door, the friend punches him in the nose.
For which of the following crimes should the friend be found guilty?
(A) Assault and battery.
(B) Battery and conspiracy to commit assault, but not assault.
(C) Battery, but not assault.
(D) Conspiracy to commit battery.
(C) Battery, but not assault.
A baseball fanatic became enraged when a fan for the other team reached out into the field of play and touched a ball, causing the winning run to score. As the spectators were leaving the stadium, the fanatic saw the other fan, who had touched the ball, walking to his car. The fanatic walked up behind the other fan and punched him in the back of the head. The other fan, who had been drinking all day, was immediately knocked out and died as soon as his head hit the pavement. This jurisdiction defines assault as: (1) the causing of physical harm to another (categorized as felony assault); or (2) acting in a threatening manner to put another in fear of immediate harm (categorized as misdemeanor assault).The fanatic was arrested for homicide of the other fan.
Which of the following is the most serious crime for which the fanatic should be found guilty?
A Assault.
B Voluntary manslaughter.
C Involuntary manslaughter.
D Felony murder.
The correct answer is: Involuntary manslaughter.
Discussion of correct answer: As a general rule, whenever an intentional battery or assault results in an unintended death, the defendant is guilty of involuntary manslaughter. This is a good example of misdemeanor manslaughter.
A gambler just gambled away her last nickel. Desperate to get her hands on some cash, the gambler decides to rob the local ice cream parlor. In preparation, she places a cigarette lighter shaped like a small gun in her coat pocket. Before entering the store, she sees that there is no one in the store other than the clerk, who is on the telephone. The gambler enters the store and tries to attract the clerk’s attention, but the clerk, engrossed in her telephone conversation, ignores the gambler. After several minutes of pacing back and forth, the gambler waves the cigarette lighter in the air and shouts, “Hey! This is a stick-up! Give me the money or I’ll shoot!” The clerk looks over her shoulder at the gambler and says, “Yeah, what is it? Oh, go ahead, help yourself. The owner, that jerk, just fired me for talking on the phone too much. I couldn’t care less.” The clerk then turns away and resumes her conversation. The gambler cleans out the cash register drawer and runs away. Of which crime is the gambler most likely to be convicted?
Attempted robbery.
After a water pipe broke in her office, a bookkeeper was sent home early from work one day. When she got there, she found her husband in a passionate embrace with a neighbor. She ran out of the house and got into her car. She phoned a pizza parlor to order a delivery, then returned to the house with the pistol she kept in her car. Two minutes after her discovery, she shot her husband and the neighbor to death.
Will the bookkeeper be found guilty of voluntary manslaughter instead of murder?
A Yes, because she was adequately provoked.
B Yes, because there was a causal connection between what the bookkeeper saw and the homicide she committed.
C No, because a reasonable person would have cooled off in two minutes.
D No, because she had cooled off by the time she reentered the house.
The correct answer is: No, because she had cooled off by the time she reentered the house.
Discussion of correct answer: Finding one’s spouse involved in sexual conduct with another is often considered to be adequate provocation for voluntary manslaughter, which is commonly called a “heat of passion” killing. It is true that, if a reasonable person would have cooled down in the time elapsed, a court is unlikely to find a defendant guilty of voluntary manslaughter. It is also true that, even if a reasonable person wouldn’t have, the mitigation to manslaughter will also not be available if the defendant actually did cool down. This crime is more likely to be murder because the bookkeeper cooled down between the discovery and the killing, as is shown by her ordering pizza and going to her car for her gun.
A police officer pulled over a car after observing it drive through a stop sign. The officer walked up to the driver’s side of the car, ordered the driver out of the vehicle, and told him to place his hands on the trunk of the car. As the driver was getting out of the car, the officer noticed a gun in his waistband. The driver stated that he did not have a permit for the gun. The officer then ordered the passenger in the front seat to also get out of the car and place his hands on the trunk. As the passenger exited the car, he dropped a small bag of heroin on the ground.
The district attorney charged the driver with illegal possession of a firearm and the passenger with illegal possession of a controlled substance. The driver filed a motion to suppress the gun and the passenger filed a motion to suppress the bag of heroin.
How should the court rule on the motions?
(A) Grant the driver’s motion, but deny the passenger’s motion.
(B) Grant the passenger’s motion, but deny the driver’s motion.
(C) Grant both motions.
(D) Deny both motions.
(D) Deny both motions.
Police pulled over a speeding car matching the description of the car driven by a kidnapper of a small child. The child was later found dead in a cornfield. The officer observed bloody clothing on the floor near the backseat and arrested the driver for kidnapping and murder. On the way to the police station, the driver blurted out that he had kidnapped the child but said that the child was still alive and that he could take the officers to the child’s location. The officer immediately read the driver his Miranda warnings and asked him if he would write the confession down at the station. The driver agreed to make another oral statement at the station but was unwilling to write anything down without a lawyer being present. The driver was then told that the child had died. The driver gave another oral confession after arriving at the police station and admitted to killing the child and lying in his first confession.
The driver’s attorney filed a motion to suppress the police car confession and the station house confession.
How should the court rule?
(A) Grant the motion as to the police car confession only, because the driver had not been read his Miranda rights.
(B) Grant both motions, because the driver had a right to counsel before making either statement.
(C) Deny the motion as to the station house confession only, because the driver had been read his Miranda warnings.
(D) Deny both motions, because both confessions were admissible as evidence.
(D) Deny both motions, because both confessions were admissible as evidence.
A breeder and owner of vicious guard dogs trained his dogs to attack strangers at night. He often sold and leased his guard dogs to various business and factory owners who used the guard dogs to frighten away intruders from entering their premises at night. One evening, the breeder was in the back yard of his home training three of his guard dogs. The back yard was enclosed with a chain link fence and a latched gate that prevented the dogs from running out. After the training session, the breeder opened the gate and permitted the dogs to run loose in his front yard. Minutes later, a man was walking along the sidewalk in front of the breeder’s house when he was attacked by one of the dogs. The man suffered severe injuries and died as a result of the attack.
The breeder should be found guilty of what crime?
A Murder.
B Involuntary manslaughter.
C Voluntary manslaughter.
D Reckless endangerment.
The correct answer is: Murder.
Discussion of correct answer: According to LaFave, “extremely negligent conduct, which creates what a reasonable man would realize to be not only an unjustifiable but also a very high degree of risk of death or serious bodily injury to another though unaccompanied by any intent to kill or do serious bodily injury and which actually causes the death of another” constitutes depraved-heart murder. See Criminal Law, p. 541. Since the breeder had trained his guard dogs to attack at night and then opened the gate to let them run loose, he will be criminally responsible for the killing of the man. Such conduct on the breeder’s part is more extreme than the gross or criminal negligence standard sufficient for involuntary manslaughter. The breeder will be liable on the theory of depraved-heart murder.
A driver was prosecuted for violation of a state statute that defines as a felony “the taking or accepting of any property, money or services by a state driver’s license examiner from or on behalf of an examinee being tested for driving proficiency.” Testimony at trial established that the driver was taking his driving test from a state driver’s license examiner when he offered the driver’s license examiner $1,000 if the examiner would overlook an illegal lane change the driver had just made (which would disqualify him from obtaining a driver’s license). The driver’s license examiner accepted the $1,000, and was subsequently convicted of violating the same statute for which the driver was being prosecuted as an aider and abettor.
What is the driver’s best argument for a dismissal of the charge?
A Only a driver’s license examiner can commit the crime defined by the subject statute.
B He cannot be convicted of committing a crime as to which he victim.
C The statute is so defined as to indicate that the legislature intended only the recipient of the property, money, or services to be punished.
D He did not assist the driver’s license examiner in violating the subject statute.
The correct answer is: The statute is so defined as to indicate that the legislature intended only the recipient of the property, money, or services to be punished.
Discussion of correct answer: Based on the given wording from the statute, only the recipient and not the giver appears to be within the intended scope of that statute. Where a crime is based upon a transaction necessarily involving at least two people, and the legislature includes only one of them within the operation of the statute, this is an indication that the legislature intended that the other person (or class of persons) not be punished by the statute. The statute at issue mentions only the recipient of the property, money, or services–the driver’s license examiner. The driver’s best argument is that he is outside the statute’s scope.
A defendant was represented by a public defender. A jury found the defendant guilty of murder. During the sentencing phase, the defendant’s attorney failed to present relevant evidence of the defendant’s upbringing that would constitute mitigating circumstances. The State presented four aggravating circumstances. The only evidence presented by the defendant’s attorney was the testimony of the defendant’s sister and his best friend, both of whom testified that the defendant was a decent man. The jury recommended the death penalty, which the judge imposed. On appeal with new counsel, the defendant contended that he had received ineffective assistance of counsel during the sentencing phase.
How should the appellate court rule?
A In favor of the defendant, because he established particular allegations of specific attorney errors.
B In favor of the defendant, because the attorney failed to investigate and present evidence that would constitute mitigating circumstances.
C Against the defendant, because a claim of ineffective assistance of counsel may only be raised in a collateral attack.
D Against the defendant, because strategic decisions of an attorney are not open to attack.
The correct answer is: In favor of the defendant, because the attorney failed to investigate and present evidence that would constitute mitigating circumstances.
Discussion of correct answer: While it is true that, in general, the law presumes that legal counsel is effective and it is the duty of the defendant to demonstrate otherwise, there are exceptions under certain circumstances. The facts of this question closely mirror the U.S. Supreme Court case of Wiggins v. Smith [539 U.S. 510 (2003)], where it was held that in a capital punishment case, trial counsel’s failure to investigate the accused’s background and to present mitigating evidence of the accused’s unfortunate life history at the sentencing proceedings violated the accused’s Sixth Amendment right to the effective assistance of counsel. Thus, given that the facts of this question closely track the compelling evidence that was before the Supreme Court in Wiggins v. Smith, this is the best answer. The appellate court should find in favor of the defendant.
A defendant had an argument with his neighbor. As they were quarreling, the defendant pulled out his penknife intending only to frighten the neighbor. The defendant accidentally slightly nicked the neighbor’s arm with the knife. Unknown to the defendant, his neighbor was a hemophiliac who then died from the cut. What is the most serious crime that the defendant can be convicted of?
a. Murder.
b. Voluntary manslaughter.
c. Involuntary manslaughter.
d. Battery
c. Involuntary manslaughter.
Police detectives looking for a gun used in the murder of a federal judge received a tip that the gun had been hidden in a storage unit from the manager of the storage unit who had seen the suspect placing a gun in the unit. The suspect was subsequently arrested and jailed on an unrelated charge. While the suspect was in jail, the detectives applied for a warrant to search the storage unit for the gun. Before they could execute the warrant, the storage unit manager told a policeman who was on patrol in the neighborhood about the gun. The patrolling officer broke into the storage unit and discovered the murder weapon. Just after the patrolling officer left the storage unit, the detectives arrived at the storage unit with their valid warrant. Subsequent ballistics testing confirmed that the gun was in fact the murder weapon, and the suspect was charged with murder. At trial, the defendant seeks to have the gun excluded from evidence.
Should the court admit the gun into evidence?
(A) Yes, because the patrolling officer’s search was reasonable due to exigent circumstances.
(B) Yes, because of the inevitable discovery rule.
(C) No, because there were no exigent circumstances justifying the patrolling officer’s search.
(D) No, because the patrolling officer’s search was unreasonable.
(B) Yes, because of the inevitable discovery rule.
A man decided to take his own life and went to the roof to jump to the street below. On his way up the stairs, he told his neighbor what he was planning to do. The neighbor had a strong belief that anyone who committed suicide would not enter Heaven. Therefore, he snuck up behind the man as he was readying to jump off the rooftop. After the man jumped, the neighbor shot him once in the back before the man struck the sidewalk below.
At trial, the medical examiner testified that the man died of a gunshot wound, and that in all likelihood, the fall would not have killed the man. The jury also found that the neighbor’s religious belief was honestly held.
If the neighbor is prosecuted for murder, what should the result be at trial?
A The neighbor should be convicted, regardless of the man’s intent or the medical examiner’s testimony.
B The neighbor should be convicted, because the medical examiner testified that the fall would not have killed the man.
C The neighbor should be acquitted, because the man intended to kill himself.
D The neighbor should be found guilty of a lesser charge, because the jury believed that his religious belief was honestly held.
The correct answer is: The neighbor should be convicted, regardless of the man’s intent or the medical examiner’s testimony.
Discussion of correct answer: Murder requires malice. Malice can be express or implied. Here, there is express malice, because the defendant intentionally took the life of another. Regardless of whether the man would have died or not otherwise is irrelevant, because it was the neighbor’s act that caused the man’s death. Moreover, the fact that the neighbor was acting in what he thought was the best interest of the victim is also irrelevant.
An accountant and a banker were leaving the gym where they played handball when a young woman confronted them. She held a large clutch purse in her left hand; her right hand was inside the purse. “I have a pistol in my purse,” she said to the accountant. She
pointed the purse at the banker and said to the accountant, “Give me your watch, rings, and wallet or I’ll kill your friend.” The accountant immediately complied, dropping all of his jewelry plus his wallet into her purse, which she held so that he could not see
inside. As the woman turned to leave, a police officer on walking patrol rounded the corner. The accountant and the banker both called for help, and the officer arrested the woman. A search of her purse for weapons revealed that she had no gun or any other
weapon.
If the young woman is prosecuted for robbery, what should be the result?
(A) She should be convicted of robbing the accountant, because she took the accountant’s property from him by threat of force.
(B) She should be convicted of robbing the banker, because the taking of the accountant’s property is considered as taking from the banker’s “person or presence.”
(C) She should be found not guilty, because she did not threaten the accountant, whose property she took, and did not take property from the banker, whom she threatened.
(D) She should be found not guilty, because she had no gun or other weapon with which to effectuate her threat of force.
(A) She should be convicted of robbing the accountant, because she took the accountant’s property from him by threat of force.
In a jurisdiction that statutorily defines burglary as the entering of any structure with intent to commit a felony therein (but otherwise follows the common law as to all crimes), a painter, an electrician, and a plumber were prosecuted for conspiracy to commit burglary. Each was tried separately. At each trial, it was established that the three had met at the painter’s house, and agreed to go to the local jewelry shop the next evening, break into the closed shop, and take a large uncut diamond from the safe. Each man then left the painter’s house to obtain dark clothing and the tools necessary for completing his part of the plan. As prearranged, the three met at the plumber’s house a few hours later and secreted their clothing and equipment in his garage. After leaving the plumber’s house, the electrician went directly to the police and informed them of the group’s activities and intentions. All three were then arrested.
The plumber testified at each trial that the painter had informed him, and he believed, that the large uncut diamond was the property of the painter, which the painter had taken to the jeweler for appraisal, and that the jeweler refused to return the diamond or to acknowledge the painter’s ownership. The plumber believed that by assisting the painter, he was merely recovering the painter’s diamond.
Assuming that the jury determines that the plumber testified truthfully, what is the result for the plumber?
A Convicted, because there was an agreement for an unlawful purpose, and an overt act in the preparation of the dark clothing and equipment sufficient to constitute a conspiracy.
B Convicted, because criminal activity undertaken with laudable motives is still criminal.
C Acquitted, because he lacked the corrupt motive necessary for conviction of conspiracy.
D Acquitted, because he believed he was agreeing to recover the painter’s diamond.
The correct answer is: Acquitted, because he believed he was agreeing to recover the painter’s diamond.
Discussion of correct answer: To be guilty of conspiracy, one must agree to achieve some unlawful purpose. If the goal of the conspiracy is a crime, the conspirators must have the same criminal intent as is necessary for conviction of the target offense. If the plumber’s testimony is truthful, he did not intend to steal; he was trying to help the painter retrieve his own diamond, which the jeweler was improperly keeping from the painter. The plumber’s mistake would negate the specific intent to steal, which is necessary both for burglary of the jewelry store and for conspiracy to commit that offense.
A man and his roommate got into an argument over a loan the man made to the roommate. The roommate called the man a spoiled brat who always got his way. The man slapped the roommate across the face. The roommate then grabbed a carving knife from the kitchen counter and tried to stab the man. The man wrestled the knife away and stabbed the roommate in the chest, killing him.
If the man is charged with homicide, which of the following statements is most accurate?
A The man is guilty of murder, because deadly force was not justified.
B The man is guilty of manslaughter, because deadly force was not justified.
C The man is guilty of murder, because he was the initial aggressor.
D The man is not guilty of any crime, because deadly force was justified.
The correct answer is: The man is not guilty of any crime, because deadly force was justified.
Discussion of correct answer: The man provoked the attack by slapping the roommate across the face. However, the slap was nondeadly force. In a majority of jurisdictions, the roommate cannot respond to nondeadly force with deadly force, such as by using a knife. The roommate used excessive force in his response to the man’s slap, and so the man can use deadly force to defend himself, and will not be responsible for criminal homicide under these circumstances.
The defendant spent the day shopping and returned to his own home just in time to see his wife half-naked and his wife’s friend jumping out the kitchen window. The defendant immediately realized that his wife had been having an affair.
Infuriated, the defendant grabbed his gun and shot and killed his wife’s friend before he reached the edge of the property. A landscaper that was outside ran over to help the wife’s friend. Suddenly, the defendant pointed the gun at the landscaper and shot him, too. The landscaper died from the gunshot wound. The wife’s friend managed to survive his injury.
The defendant was prosecuted for the homicide of the landscaper. The jury found that the defendant was reasonably and subjectively provoked by the thought of his wife having an affair.
For what crime should the defendant be found guilty?
A No crime.
B Involuntary manslaughter.
C Voluntary manslaughter.
D Murder.
The correct answer is: Murder.
Discussion of correct answer: An intent-to-kill homicide that would otherwise be murder can be mitigated to voluntary manslaughter if the victim has provoked the attack in such a manner that the defendant reasonably and subjectively lost his self-control. Observing one’s spouse having sex with another person is just such a mitigating factor. However, where the defendant intentionally kills someone other than the provocateur, is the “heat of passion” engendered by the provocation extended to mitigate the homicide of the innocent third party. Generally, the answer is no; even if a sufficient provocation exists to mitigate the intentional killing of the provocateur, the intentional killing of an innocent third party is not subject to the same mitigation, and if no other justifying, excusing, or mitigating factor is present, the killing of that third party is considered murder. Because the defendant killed an innocent third party here, he should be found guilty of murder.
A witness was subpoenaed to testify in front of a federal grand jury regarding drug trafficking. The witness took the stand and was given his Fifth Amendment rights and told he did not have to testify if he chose not to. The witness asked to have his attorney present inside the grand jury room. The prosecutor denied this request. The witness was allowed to speak with his attorney outside the grand jury room and elected to testify. The witness testified and incriminated himself in a drug trafficking conspiracy. The witness was indicted based on his grand jury testimony and evidence obtained from an illegal search of the witness’s home. The witness’s attorney moved to dismiss the indictment.
Should the court dismiss the indictment?
A Yes, because he was denied his constitutional right to an attorney.
B Yes, because the evidence against him was illegally obtained.
C No, because the witness waived his constitutional rights by testifying.
D No, because the witness had no right to an attorney inside the grand jury room, and the illegally seized evidence did not taint the validity of the indictment.
The correct answer is: No, because the witness had no right to an attorney inside the grand jury room, and the illegally seized evidence did not taint the validity of the indictment.
Discussion of correct answer: The witness had no right to an attorney inside the grand jury room but did have a right to consult with counsel outside the grand jury room which he exercised. The Fourth Amendment exclusionary rule does not apply to grand jury proceedings so the illegally seized evidence would not taint the indictment itself.
A mother and father were members of a religious sect that believed in the power of faith healing. They lived in a state where the common-law rules concerning murder have not been altered by statute or decision. When their six-year-old daughter broke her leg, the mother and father had their pastor come over to give his opinion. He believed that prayer would heal the daughter and told the mother and father not to take the daughter to the hospital. They agreed and kept her at home. The daughter developed a fever and died three days later. The mother and father have been indicted for the crime of murder.
If they are acquitted, what would be the best reason?
A They did not intend to kill or to harm the daughter.
B They neither premeditated nor deliberated.
C They in good faith relied upon what the pastor told them, so if the pastor was wrong, they have the defense of mistake of fact.
D The First Amendment Free Exercise Clause protects their sincerely held religious beliefs.
The correct answer is: They did not intend to kill or to harm the daughter.
Discussion of correct answer: A murder conviction requires an intentional killing, or the intentional infliction of serious bodily harm. If the defendants neither intended to harm the daughter, nor to kill her, they are not guilty of murder. It is clear from the facts of the question that they did not desire the death of their daughter, nor did they desire to do her great bodily harm. Therefore, if they are acquitted, it will be because they did not have the requisite intent to commit common-law murder.
Two men, one five years older than the other, were rivals for the same promotion. The older man knew that the younger man’s spouse was a very jealous person with a sometimes uncontrollable temper. The older man decided to eliminate the younger man as a competitor for the promotion by prodding the younger man’s wife into killing him. One day, at 11:45 a.m., the older man disguised his voice and called the younger man’s wife, telling her that her husband had been having an affair with a co-worker for several weeks and that they met every day at noon in the company storage room. The older man knew this was not true, but he hoped this news would send the younger man’s wife into a jealous rage. The wife, who worked in an adjoining building, immediately rushed to the company at which her husband worked and waited out of sight in the storage room. Enraged, she brought with her a gun she kept in her desk. At noon the older man asked the younger man to go to the storage room and retrieve some supplies needed in the ofc. When the younger man entered the storage room, his wife, still in a jealous rage, pulled out the gun and shot and killed her husband. The older man was prosecuted for the homicide of the younger manFor which crime should the defendant be found guilty
First-degree murder.
In which of the following instances is the defendant most likely to be found guilty of common law
arson?
(A) A teenager goes to his ex-girlfriend’s house. He gathers up her extensive collection of stuffed animals, places them in a pile in the bathtub, and sets the pile on fire. He then flees the area. The stuffed animals are mostly new and made of flame-retardant materials, and the fire burns out fairly quickly, only damaging some of the stuffed animals.
(B) A bouncer hates a bartender because he is always showing off. The bouncer goes to the bartender’s house and throws a firebomb into the bartender’s brand new luxury car, completely destroying it.
(C) A babysitter is 18 years old. He is bored one afternoon and notices that there is a huge pile of leaves next to his neighbor’s house. The babysitter walks over and sets the leaves on fire, because he figures seeing the fire will be exciting. Once they are ignited, however, the babysitter realizes that the process is not nearly as exciting as he had expected, so he leaves. Eventually, the fire from the burning leaves spreads to the house, destroying the family room of the house.
(D) A senator has a very romantic night planned for his girlfriend. He goes to her house and starts setting the scene. He lights numerous candles and places them in various locations around the house. Realizing that he forgot to bring the Belgian chocolates and champagne, he runs out to buy them. While he is gone, his girlfriend’s cat knocks over a candle, and the house catches fire. By the time the senator returns 10 minutes later, the whole house is engulfed in flames.
(C) A babysitter is 18 years old. He is bored one afternoon and notices that there is a huge pile of leaves next to his neighbor’s house. The babysitter walks over and sets the leaves on fire, because he figures seeing the fire will be exciting. Once they are ignited, however, the babysitter realizes that the process is not nearly as exciting as he had expected, so he leaves. Eventually, the fire from the burning leaves spreads to the house, destroying the family room of the house.
A defendant hated his boss, who had recently demoted him to a less prestigious position. Late one afternoon, the defendant saw his boss walking down the hallway. The defendant pulled out a gun and fired four shots at his boss. Although none of the bullets directly hit his boss, one of the shots ricocheted against a wall and struck the boss in the head, killing him instantly. What is the most serious crime that the defendant can be convicted of?
a. Murder.
b. Voluntary manslaughter.
c. Involuntary manslaughter.
d. battery.
a. Murder.
A woman waited on a train platform to take the next train home. A man walked up beside her and said, “I haven’t eaten in three days. Can you please give me some help?” The man had a gun in his pocket but did not show it to the woman. The woman became terrified because of all of the violent crime known to happen on that train platform. Fright-stricken, the woman reached into her purse and gave the man $20. The man took the money and casually walked off into the crowd.
Was the woman the victim of a robbery?
A Yes, because the man was armed.
B Yes, because the woman was frightened.
C No, because the woman did not know about the gun.
D No, because the man did not use violence.
The correct answer is: No, because the woman did not know about the gun.
Discussion of correct answer: A robbery occurs when there is an unlawful taking and carrying away of the personal property of another and the taking is from the victim’s person or in their presence and force or intimidation is used. The force or intimidation does not have to be realistic, e.g. an unloaded gun is sufficient to establish the element of force. However, the victim does have to be aware of the force being used or threatened. Here, the woman’s fear was the result of factors outside of the exchange between her and the man–she was scared of being in a high crime area. Although the man was armed, he asked for money in a non-threatening way (he explained his situation and said “please”) and did not show the woman the gun. Thus, even though she was scared, her fear was not caused by the man in the course of him taking something from her. Since the woman did not surrender the money in response to the use of force or intimidation, no robbery occurred. Test tip: Ask yourself what the defendant’s best defense would be, and if it would cancel out or negate the mens rea or actus reus of the crime he is charged with. In this case, he never threatened the woman. He is not guilty, because this defense cancels out the element of robbery that requires that the taking be from the victim’s person or in his/her presence and that force or intimidation is used.
The police received a tip that a man was selling drugs on a street corner. Two officers parked their unmarked car on the opposite side of the street and began to observe the man. He was approached several times by individuals and, after receiving something from them, would reach into his pocket and hand something back, at which point they would walk away.
As a marked police car came down the block, the man dashed into his pickup truck to escape. Unable to start the truck, he fled across a vacant parking lot towards a small shopping mall. An officer tackled him just as he entered the parking lot. As the man attempted to get up, the officer noticed that a bag containing cocaine was lying on the ground underneath the man. The man was arrested and charged. At trial, the man moved to suppress the cocaine.
How should the court rule?
A Grant the motion, because the police had not obtained a warrant.
B Grant the motion, because the police lacked probable cause.
C Deny the motion, because the police had probable cause to arrest the man.
D Deny the motion, because the drugs were in plain view.
The correct answer is: Deny the motion, because the police had probable cause to arrest the man.
Discussion of correct answer: An individual has the right to be free of unreasonable searches and seizures under the Fourth Amendment. However, the police may acquire probable cause from any evidence that they observe. Here, based on the totality of the circumstances, the police had probable cause to arrest the man. Note that answer choice (D) is incorrect because it does not account for how the police were at a lawful vantage point when they observed the bag containing the white powder.
To investigate a suspected large-scale marijuana farming operation, police officers used a helicopter to fly over an isolated and unoccupied large garden shed. The police used a high-resolution heat-sensing camera that is not available for sale to the public to take thermal photographs of the outside of the shed that suggested the presence of heat lamps associated with marijuana cultivation. The police did not enter the shed, but at trial sought to introduce the thermal pictures as part of their evidence of defendant’s drug production. Defendant seeks to have the evidence excluded.
Should the court admit the evidence?
A Yes, because of the plain view doctrine.
B Yes, because the shed was not within the curtilage of a dwelling.
C No, because the camera was not generally available to the public.
D No, because the defendant had a reasonable expectation of privacy in the shed.
The correct answer is: Yes, because the shed was not within the curtilage of a dwelling.
Discussion of correct answer: The Fourth Amendment gives a reasonable expectation of privacy to a dwelling, as well as outbuildings within the curtilage of the dwelling, the living space immediately outside the dwelling. However, under the open fields doctrine, areas beyond the curtilage do not receive Fourth Amendment protection. Because the shed was isolated and far away from the dwelling, the police thermal scanning does not violate the Fourth Amendment, so the exclusionary rule does not apply.
An elderly couple was involved in a small car accident. While both the husband and wife seemed to be uninjured, medical tests indicated the husband had suffered internal bleeding and needed surgery to repair the damage. The husband was placed on life support while he awaited the surgery. A doctor told the couple there was likely only a 10% chance the husband would survive the surgery. The husband slipped into a coma shortly thereafter. The wife recalled the husband telling her that his worst fear was to die on an operating table. After consulting with the couple’s son, the wife turned off the machine providing life support to her husband, and he promptly died. An autopsy revealed that the husband’s injuries were so severe that he would not have survived the surgery.
At common law, what is the most serious crime for which the wife may be convicted?
A Involuntary manslaughter.
B Voluntary manslaughter.
C Depraved-heart murder.
D Intentional murder.
The correct answer is: Intentional murder.
Discussion of correct answer: Murder is defined as the unlawful killing of a human being with malice aforethought. In situations when a victim is already dying, if the defendant’s actions bring about the victim’s death more quickly than if the defendant had not acted, the defendant’s actions would be an actual cause of the killing. While the autopsy indicated that the husband would not have survived the surgery, the wife is nonetheless guilty of intentional murder for taking an action would ended the husband’s life. The fact that the husband had previously stated he feared dying on an operating table does not provide a defense.
A bookstore owner was believed to be responsible for the arson of a competing bookstore. Police brought her to the police station for questioning but had not placed her under arrest. A detective sat down with the bookstore owner, read her the Miranda warning and bluntly asked why she did it. The bookstore owner said, “I know I don’t have to talk to you. I’m invoking my right to silence.” The detective stopped talking to her and walked away. Thirty minutes later, the detective asked the bookstore owner if she wanted some water, to which the bookstore owner answered, “Yes, nothing makes you thirsty like being around a bunch of burning books.” The detective then reads her the Miranda warning.
If the bookstore owner’s statement is admitted in a trial for arson, what is the most likely reason?
(A) The bookstore owner waived her Miranda rights.
(B) The bookstore owner did not have to be Mirandized a second time for the officer to asks her questions.
(C) The bookstore owner made a voluntary statement.
(D) The bookstore owner was not in custody.
(A) The bookstore owner waived her Miranda rights.
A police officer followed a car with expired tags and pulled the car over. Before approaching the car, the officer checked the license plate against a database and found that the owner of the car was years behind in his child support and there was a warrant for his arrest. The officer approached the car, and determined the driver was the owner. He asked the driver to step out of the car, arrested him and handcuffed him, and then placed the driver in the back of the police car. The officer did a quick search of the car limited to the driver’s wingspan, and found an envelope containing cocaine in the glove compartment. The driver was then arrested for drug possession. At trial, he made a motion to suppress the introduction of the cocaine.
Will the court grant the motion?
A Yes, because the officer’s search of the car violated his Fourth Amendment protections.
B Yes, because the officer lacked probable cause to arrest the driver.
C No, because the seizure of the cocaine was the product of a valid search incident to a lawful arrest.
D No, because probable cause allows an officer to search within containers found within an automobile.
The correct answer is: Yes, because the officer’s search of the car violated his Fourth Amendment protections.
Discussion of correct answer: While an officer may conduct a warrantless search of an automobile incident to a lawful arrest, this may only be done when it is reasonable to believe the defendant might access the vehicle at the time of the search or that the vehicle contains evidence of the offense of arrest. Here, the driver was handcuffed and in the back of the police car at the time of the arrest, and his arrest for child support was completely unrelated to the officer’s search. As a test tip, it is helpful to think of policy considerations when deciding if a probable cause exception to the warrant requirement should apply. The purpose of the police being able to conduct a wingspan search incident to a lawful arrest is to promote the safety of the policeman. Once the defendant is handcuffed, the policeman is no longer in danger and a wingspan search is no longer needed or lawful.
A woman was driving her van along a public road one night. A police officer who was driving behind the woman, decided to make a random stop of the woman’s vehicle to check her license and registration. The officer pulled the woman’s van over to the side of the road and then walked up to the driver’s side of the vehicle. When he came alongside the driver’s window, the officer asked the woman for her identification. As the woman was thumbing through her wallet, the officer shone his flashlight into the van and spotted a plastic bag containing marijuana lying on the floor under the back seat. The officer then arrested the woman and charged her with possession of marijuana. At the woman’s trial for illegal possession of a controlled substance, her attorney moved to suppress the use of the marijuana as evidence.
How should the judge rule on her motion?
(A) Granted, because the marijuana was the fruit of an illegal search.
(B) Granted, because the police officer did not have probable cause or a reasonable suspicion to believe that the woman’s van contained a controlled substance.
(C) Denied, because the marijuana was in plain view when the police officer shone his flashlight inside the van.
(D) Denied, because the marijuana was in an automobile.
(A) Granted, because the marijuana was the fruit of an illegal search.
In which of the following circumstances will intoxication least likely serve as a valid defense to the crime charged?
A A defendant becomes intoxicated after drinking a beverage that someone had spiked with a powerful drug, without his knowledge. Afterwards, he has sexual relations with a minor. Due to the effect of the drug, he had mistaken the minor for his adult girlfriend. He is charged with statutory rape.
B A defendant was at a friend’s party and had too much to drink when she decided to show off a new “magic trick” she had been working on. Her trick involved setting the pages of a book on fire, but, due to her inebriation, she performed the trick incorrectly and set a fire causing significant damage to the friend’s basement. She is charged with arson.
C A defendant became drunk one night, and on his way home, took a wrong direction and found his way to an apartment that he mistakenly thought was his own. Finding the door locked, he shoved the door until it busted open. He went inside, and the frightened residents called the police. He is charged with burglary.
D A defendant ordered a non-alcoholic drink but was served an alcoholic drink by the bartender instead. The defendant, who was unaware of the alcohol, had a severe reaction to the alcohol and, during his conversation with the bartender, came to the conclusion that the bartender was a government agent sent to eradicate him. He punched the bartender in the face and ran out of the bar. He is charged with battery.
The correct answer is: A defendant was at a friend’s party and had too much to drink when she decided to show off a new “magic trick” she had been working on. Her trick involved setting the pages of a book on fire, but, due to her inebriation, she performed the trick incorrectly and set a fire causing significant damage to the friend’s basement. She is charged with arson.
Discussion of correct answer: Voluntary intoxication may be a valid defense for a specific intent crime if it negates the requisite mental state, but it will not negate recklessness, negligence or strict liability. Arson requires only recklessness, so voluntary intoxication will not be a valid defense to arson. Test tip: Involuntary intoxication is always a good defense. Voluntary intoxication is sometimes a good defense, and sometimes it is not. When the defendant gets so drunk that the intoxication mistakenly causes him to think he had a right to do something and was not doing something illegal when in fact he was, it is a good defense. That belief would negate the specific intent to commit a crime, but would not negate having the malice mens rea (recklessness) for crimes such as arson.
The victim owned and operated a small store. The victim also lived in an apartment above the store. Late one evening, the defendant decided to break into the store to steal beer. The defendant threw a brick through the window of the store and went inside. Awakened by the alarm, the victim went down the stairs and into the store. Seeing the victim, the defendant turned and began to run. The victim ran after the defendant but tripped and fell, breaking his neck, which resulted in his immediate death. The defendant was subsequently charged with the victim’s death. Of what crime, if any, could the defendant be found guilty?
Felony murder.
Under which of the following fact situations would the defendant’s Miranda waiver most likely be ineffective?
(A) The defendant recently graduated from law school. At her graduation party, the defendant became highly intoxicated. Following the party, the defendant drove home in her automobile. She fell asleep at the wheel and crashed into another vehicle, injuring the driver. Shortly after the accident, a police officer came on the scene and arrested the defendant, charging her with D.U.I. The defendant was then given her Miranda warnings and transported to the police station. Upon questioning, the defendant–who was still highly intoxicated–waived her Miranda rights and made an incriminating statement.
(B) The defendant stabbed a victim after a violent argument. Following the victim’s death, the police arrested the defendant and charged him with murder. He was then transported to the station house, where Miranda warnings were given. Afterwards, the defendant was interrogated and proceeded to waive his Miranda rights. He then confessed to committing the crime. At trial, a psychiatrist testified that the defendant was mentally ill and his confession was not the result of a knowing and intelligent waiver.
(C) The defendant was a 15-year-old high school sophomore. He possessed normal intelligence and experience for a youth of his age. One night, he and two friends attended a concert. After the concert, the defendant and his friends went on a spree, assaulting and robbing victims in the park. The next day, the defendant was arrested. After being subjected to persistent questioning for two hours, the defendant was first given his Miranda warnings. The defendant then waived his Miranda rights and offered a confession. At trial, the defendant claims he did not make a knowing and intelligent waiver.
(D) The defendant was a 16-year-old juvenile in police custody on suspicion of murder. He had been on probation for a series of juvenile offenses. After he was given his Miranda warnings, he requested to have his probation officer present. His request was denied. During a brief interrogation, the defendant proceeded to waive his Miranda rights and made incriminating statements that linked him with the crime. At trial, the defendant’s lawyer claims that his waiver was ineffective because his request to see the probation officer was the equivalent of asking for a lawyer.
(C) The defendant was a 15-year-old high school sophomore. He possessed normal intelligence and experience for a youth of his age. One night, he and two friends attended a concert. After the concert, the defendant and his friends went on a spree, assaulting and robbing victims in the park. The next day, the defendant was arrested. After being subjected to persistent questioning for two hours, the defendant was first given his Miranda warnings. The defendant then waived his Miranda rights and offered a confession. At trial, the defendant claims he did not make a knowing and intelligent waiver.
A husband and wife had been married for many years. Normally, the husband stayed at work each weekday, returning home after 6 p.m. The wife worked from their home as a telemarketer. As their 30th anniversary approached, the husband decides to prepare a romantic surprise for his wife. On the day of their anniversary, the husband left work at 1 p.m. and drove to the local florist, where he purchased a dozen long-stemmed roses. He then drove home, intending to surprise his wife with the flowers and spend the entire afternoon alone with her. When the husband arrived at his home, he looked in the window of the office his wife used to make her telemarketing calls and observed his wife in what he believed to be a passionate, sexually charged embrace with their neighbor. The husband, a jealous man, had long suspected that the neighbor harbored desires for his wife. Believing that his wife and the neighbor were engaged in sexual relations, the husband flew into a rage, ran to the garage, and retrieved a loaded revolver he kept in a locked box. He then ran back to the front of the house just as the neighbor was leaving. Still in a jealous rage, the husband shot the neighbor, killing him instantly. In actuality, the neighbor had dropped by the happy couple’s home to drop off an anniversary gift for the two of them and the wife was giving their neighbor a hug when their belt buckles accidentally got hitched together, causing them to squirm awkwardly for a moment at the precise time the husband had looked through the window. The husband was prosecuted for the homicide of the neighbor. If the jury found that the husband reasonably believed that his wife and the neighbor were having sexual relations at the time of the incident, the husband should be found guilty of what crime, if any?
Voluntary manslaughter.
A police officer went undercover at a college, posing as a freshman student in order to investigate rumors of drug-making activities among graduate students in the school’s chemistry department. The officer volunteered to help a female teaching assistant clean a lab one night. While doing so, the officer noticed that the teaching assistant was stealing certain chemicals known to be ingredients in a synthetic drug that had been spreading across the campus. Believing the teaching assistant would make a good confidential informant, pulled out a pair of handcuffs, and informed the teaching assistant that he was arresting her. Before the officer could show his badge, the teaching assistant, not knowing that the person she believed to be a freshman student was actually a police officer, and who thought she was being sexually assaulted, began throwing empty beakers at the officer, hitting him in the chest and face but not injuring him. The officer was able to arrest the teaching assistant nonetheless.
If the teaching assistant is charged with battery, will she have a valid defense?
A Yes, because she was not aware that the freshman was actually a police officer.
B Yes, because a defendant may use reasonable, non-deadly force to resist an unlawful arrest.
C No, because her belief that she was being sexually assaulted was not reasonable.
D No, because she was in the process of committing a crime and thus subject to a lawful arrest.
The correct answer is: Yes, because she was not aware that the freshman was actually a police officer.
Discussion of correct answer: An individual may use reasonable, non-deadly force to resist a lawful arrest by a police officer, but only where the individual does not know that the other person is a police officer. Here, despite the fact that the teaching assistant was engaged in criminal activity, she was not aware that the freshman was a police officer, and it was reasonable to use non-deadly force to resist what she believed to be an assault.
In which of the following cases was the defendant guilty of murder?
A The victim slapped the defendant’s wife in the defendant’s presence. The defendant became enraged and killed the victim.
B The defendant observed the victim stealing the hood ornament from his car. The defendant immediately shot the victim.
C The victim, a police officer, insulted a juvenile, the defendant, age 16, by calling him a “dumb little scumbag” without justification. The defendant stabbed and killed the officer.
D The defendant and the victim had an argument in a bar. The victim threw whiskey in the defendant’s face. The defendant wiped his face, pulled out a knife, and fatally stabbed the victim.
The correct answer is: The victim, a police officer, insulted a juvenile, the defendant, age 16, by calling him a “dumb little scumbag” without justification. The defendant stabbed and killed the officer.
Discussion of correct answer: The verbal insult by the police officer is not sufficient provocation to reduce murder to manslaughter. Moreover, a juvenile, age 16, is capable of possessing the mens rea necessary for murder and may be prosecuted for that crime, though he may be prosecuted in juvenile court.
A college student was a committed vegan, eating only plant-derived foods, and the thought of all the animals slaughtered to provide meat for people nauseated her. One night, in order to dramatize the unhealthy nature of meat and to draw attention to the slaughter of innocent animals, the student went to the butcher’s section of the supermarket where she was employed as produce manager and sprinkled a nausea-inducing chemical on all the meat, fowl and fish products. Despite being careful, the student accidentally put too much powder in one package of hamburger. When the woman who bought it served it to her family, her husband, who was unusually susceptible, died as a result of ingesting the chemical. In the student’s jurisdiction, murder is defined as: “the unlawful killing of a human being with malice aforethought. Such malice may be express or implied. It is express when there is manifested a deliberate intention unlawfully to take away the life of another person. It is implied when no considerable provocation appears or when the circumstances attending the killing show an abandoned and malignant heart. All murder which is perpetrated by willful, deliberate, or premeditated killing or which is committed in the perpetration of or attempt to perpetrate arson, rape, robbery, or burglary is murder of the first degree. All other kinds of murders are of the second degree.” The student is subsequently prosecuted for the criminal homicide of the woman’s husband. Of what crime should the student should be found guilty?
Involuntary manslaughter, because she did not intend to kill anyone.
A caterer hated her assistant and decided to kill her. The caterer poured a glass of wine for the assistant that contained enough poison to kill several people. As the caterer was handing the wine to the assistant, the assistant’s mother suddenly swept into the dining room, grabbed the glass of poisoned wine from the caterer, and sipped from it. The mother immediately became ill, vomited, and was left with only a slight headache and persistent nausea. The caterer was indicted for the attempted murder of her assistant’s mother.
How should the jury find?
A Not guilty, because the mother took the wine from her hand.
B Not guilty, because she did not intend to kill the mother.
C Guilty, because she intended to kill the assistant.
D Guilty, because she planned to kill the assistant and acted “in cold blood.”
The correct answer is: Not guilty, because she did not intend to kill the mother.
Discussion of correct answer: Generally, to be guilty of an attempt, the defendant must have the specific intent to commit the target crime. Here, the caterer did not have the specific intent to murder the mother, only to murder the assistant. While this may appear to be a case where the transferred intent doctrine may apply, that doctrine does not apply to attempt crimes. Thus, the caterer will not be guilty of attempted murder of the mother.
A woman was driving her van along a public road one night. A police officer who was driving behind the woman, decided to make a random stop of the woman’s vehicle to check her license and registration. The officer pulled the woman’s van over to the side of the road and then walked up to the driver’s side of the vehicle. When he came alongside the driver’s window, the officer asked the woman for her identification. As the woman was thumbing through her wallet, the officer shone his flashlight into the van and spotted a plastic bag containing marijuana lying on the floor under the back seat. The officer then arrested the woman and charged her with possession of marijuana.
At the woman’s trial for illegal possession of a controlled substance, her attorney moved to suppress the use of the marijuana as evidence.
How should the judge rule on her motion?
A Granted, because the marijuana was the fruit of an illegal search.
B Granted, because the police officer did not have probable cause or a reasonable suspicion to believe that the woman’s van contained a controlled substance.
C Denied, because the marijuana was in plain view when the police officer shone his flashlight inside the van.
D Denied, because the marijuana was in an automobile.
The correct answer is: Granted, because the marijuana was the fruit of an illegal search.
Discussion of correct answer: A random stop of a vehicle on the highway where the officer has no suspicion of wrongdoing is unconstitutional because it leaves too much discretion to the police officer [Delaware v. Prouse, 440 U.S. 648 (1979)]. And, applying the fruits of the poisonous tree doctrine, no evidence seized as a result of a Fourth Amendment violation may be admitted at trial [Wong Sun v. United States, 371 U.S. 471 (1963)]. Note: The rule suppressing fruits of an illegal search applies not only to objects found, but also to verbal statements obtained because of the original tainted search or as a result of an illegal arrest [Brown v. Illinois, 422 U.S. 590 (1975)].
A burglar entered a house at night by breaking a large window on the ground floor of the home. He stole a television from the house and then fled the premises, but left the large window broken and open. An hour later, a bear that had been roaming around in the woods behind the house entered the home through the open window. When the owner of the home arrived at his house later that night, the bear attacked and killed the homeowner. The burglar was arrested and charged with the death of the homeowner. In the neighborhood where the house was located, it was not uncommon for bears to be found lurking in the woods and there had been several bear sightings in the weeks preceding the burglary.
Will the burglar be found guilty of murdering the homeowner?
A No, because the homeowner’s death was caused by an independent intervening cause that superseded the burglar’s act.
B No, because the homeowner’s death was caused by a dependent intervening cause that superseded the burglar’s act.
C Yes, because the homeowner’s death was caused by an independent intervening cause that did not supersede the burglar’s act.
D Yes, because the homeowner’s death was caused by a dependent intervening cause that did not supersede the burglar’s act.
The correct answer is: Yes, because the homeowner’s death was caused by a dependent intervening cause that did not supersede the burglar’s act.
Discussion of correct answer: The presence of the bear in the house is a dependent intervening cause of the homeowner’s death because this occurrence was the result of the burglar’s act of breaking the window and leaving it open when he entered the home. The bear would not have been in the home but for the burglar’s actions. A dependent intervening cause does not supersede the burglar’s act and relieve him from liability unless the result of his act was totally abnormal. Because bear sightings were not uncommon in the neighborhood where the burglary occurred, the bear’s presence in the home was not totally abnormal or unforeseeable, and therefore, this dependent intervening cause of the homeowner’s death does not supersede the burglar’s act to relieve him of responsibility for the homeowner’s death. As a test tip, in this case, the only type of murder he could possibly be convicted of would be felony murder. For that crime he would be liable for the death of the homeowner if the cause of the death was foreseeable and linked to something the burglar did during the commission or flight from the burglary. It does not matter that a bear entered the house one hour after the burglary was committed, because the burglar left the window open during his burglary and that made it possible for the bear to enter his home. The bear entering the home was foreseeable because there had been several bear sightings in area for the weeks preceding the burglary.
After months of heartbreaking work, a hospice worker decided to end his life by driving over a bridge one night after his shift. A teenager stayed up all night drinking at a party. The teenager got in the car to drive home. While fiddling with the radio, the teenager veered off his side of the road and into the hospice worker’s car, killing the hospice worker instantly.
Is the teenager guilty of manslaughter?
A The teenager is guilty, because there was no intervening act.
B The teenager is guilty, because intoxication is not a defense.
C The teenager is guilty, because he has the defense of intoxication.
D The teenager is not guilty, because suicide is not homicide.
The correct answer is: The teenager is guilty, because intoxication is not a defense.
Discussion of correct answer: The teenager is guilty of homicide and intoxication is not a defense. Criminal homicide is the killing of a human caused by another human. Voluntary intoxication will not negate a defendant’s mens rea or intent to kill. Voluntary intoxication is not a defense to general intent crimes. Manslaughter is a general intent crime. Here, the teenager’s drunk driving was the proximate cause of the hospice worker’s death, thus, the teenager is guilty of homicide and intoxication is not a defense. Test tip: A death caused by drunk driving is always involuntary manslaughter. When the question asks if the defendant is guilty of manslaughter, he/she is guilty if you think he/she is guilty of either involuntary or voluntary manslaughter. The victim’s desire to commit suicide is a trick. The defendant is guilty if he killed the person even one second before the victim would have died from other causes.
A pilot had loaned some money to a co-pilot at 100% per week interest, and the co-pilot failed to repay the loan. The pilot telephoned his friend’s residence, which the friend shared with his father, and left a message for his friend on the telephone answering machine telling him that the co-pilot had defaulted on the loan and that the friend should break the co-pilot’s left arm that same night, but not kill him. Unknown to the pilot, his friend was at the racetrack all day and did not return to his residence. The friend’s father heard the telephone message and, because he had offered on numerous occasions to do such work for the pilot, thought that the pilot had finally decided to give him a job as an enforcer. The friend’s father immediately began searching for the co-pilot. The friend’s father found the co-pilot selling drugs to adolescents at a shopping mall, took the co-pilot to the rear of the mall, and broke his arm by dropping a heavy cement block on it. The block crushed the co-pilot’s arm and severed a major artery, causing the co-pilot to bleed to death within a few minutes.Is the pilot guilty of soliciting his friend’s father to commit a crime?
No, because the pilot intended to instruct his friend, not the father, to break the co-pilot’s arm.
A doctor and a lawyer were customers in a pub, and late in the evening, they got into a loud shouting match. After trading a number of insults, the doctor raised his fist and shouted, “I ought to show you!” A former intern of the lawyer was a customer in the
pub. The former intern called out, “Do it! That guy needs a beating!!” The doctor then grabbed the lawyer and headbutted him in the face. The doctor was subsequently convicted of battery against the lawyer. The former intern is prosecuted for battery against the lawyer.
What should be the result after trial?
(A) Not guilty, because his words alone were not sufficient to imply any agreement between him and the doctor to attack the lawyer.
(B) Not guilty, because mere presence at the scene of a crime and oral encouragement, whether or not the former intern had the requisite intent, is not sufficient for conviction as an aider and abettor.
(C) Guilty, because he encouraged the doctor to attack the lawyer.
(D) Guilty, because the former intern’s actions created an implied agreement with the doctor to attack the lawyer.
(C) Guilty, because he encouraged the doctor to attack the lawyer.
A husband came home early from work one day. He walked to the bedroom and, upon opening the door, saw his wife and another man in bed together. Visibly upset, the husband told the other man to leave. As the man was reaching for his pants, he pulled out a gun and started to raise it towards the husband, fearing that the husband was about to attack him. When the husband saw the gun, he jumped on the man and, during the ensuing struggle, angrily twisted his neck, breaking it and killing him.
The husband is guilty for which, if any, of the following crimes?
A Murder.
B Voluntary manslaughter.
C Involuntary manslaughter.
D No crime.
The correct answer is: No crime.
Discussion of correct answer: In this question, the facts clearly indicate that the other man was the aggressor. When the husband caught the other man in bed with his wife, the husband did not threaten the other man with bodily harm or injury. It was only after the other man brandished his weapon that the husband acted in self-defense and killed him, thus he has not committed any crime.
A man took two people hostage and arranged for them to play a game. The man blindfold the first hostage and strapped a gun, which was pointed at the first hostage’s head, to the second hostage’s hand. The man told the second hostage that if he did not pull the trigger, that the man would shoot the second hostage. Not wanting to die, the second hostage pulled the trigger, killing the first hostage.
If the second hostage is charged with the death of the first hostage, how should the jury find the second hostage?
A Guilty of murder.
B Guilty of manslaughter.
C Not guilty, because the second hostage was under duress.
D Not guilty, because the killing was done out of necessity.
The correct answer is: Guilty of murder.
Discussion of correct answer: The second hostage intentionally pulled the trigger knowing that the gun was pointed at the first hostage’s head. He chose to kill rather than to be killed, and he would therefore be guilty of murder. Note that duress is a defense to everything except murder, because one person’s life is not more important than anyone else’s.
Police received information from an undercover police officer that she had just seen two men (whom she described) in a red four-door sedan selling marijuana near the city’s largest high school. A few minutes later, two police officers saw a sedan fitting the description half a block from the high school. The driver of the sedan matched the description of one of the men described by the undercover officer.
The two officers stopped the sedan and searched the driver. In the pocket of the driver’s jacket, the police found a small bottle of pills that they recognized as narcotics. They also looked in the trunk, where they found a quantity of cocaine in a sealed envelope. The driver was arrested and charged with unlawful possession of narcotics. The driver moved to suppress the use of the cocaine found in the trunk as evidence.
How should the court rule?
A Grant the motion, because the search was without probable cause or a warrant.
B Grant the motion, because the search went beyond the wingspan of the driver.
C Deny the motion, because this was a search incident to a lawful arrest.
D Deny the motion, because the police had probable cause to search the entire vehicle.
The correct answer is: Deny the motion, because the police had probable cause to search the entire vehicle.
Discussion of correct answer: A search without a warrant is valid if the search is of an automobile that is capable of being moved by its occupants and the police have probable cause to believe that it is carrying contraband, or where other exigent circumstances require an immediate search. Here, the police had probable cause to believe that drugs were being dealt from the sedan, because a few minutes earlier, an undercover police officer had identified two men in a similar car as selling marijuana near the city’s largest high school. The police stopped the defendant’s car, which fit the description, half a block from the high school. The driver of the car matched the description of one of the men described by the undercover officer. Because the requirements of the automobile warrant exception had been met, the police could search the entire car. Therefore, the cocaine found in the trunk will be admissible.
After a heated argument at work a waiter poured a glass of soda over a cook’s head. This was the final straw for the cook and he decided to get even with the waiter. The cook approached a fraternity brother, who also hated the waiter, and offered to pay the fraternity brother $5,000 to hide outside the waiter’s door and hit him in the knees with a baseball bat when he left for work in the morning. The cook made it clear he only wanted to injure the waiter, not kill him. The fraternity brother agreed. After thinking about it, the fraternity brother did not want to risk getting caught but did not want to give up the money, so he approached his roommate and offered to pay his roommate $2,500 if he would carry out the attack. The roommate agreed. The next morning the roommate waited outside the waiter’s door and hit him in the kneecaps with a baseball bat. After being hit the waiter fell back and hit his head against the door frame. He was killed instantly.
Is the cook criminally liable for the death of the waiter?
A No, because the cook did not intend the waiter’s death.
B No, because the cook’s agreement was with the fraternity brother and not the roommate.
C Yes, because the cook solicited the homicide.
D Yes, because the cook’s actions were an actual cause of the waiter’s death.
The correct answer is: Yes, because the cook’s actions were an actual cause of the waiter’s death.
Discussion of correct answer: The cook is liable for the waiter’s death in a number of ways. First, he is part of a hub-and-spoke conspiracy. Even though he did not know the roommate, the cook is liable for those who are involved in a whole conspiracy and any foreseeable outcomes. A death that results from an attack with a deadly weapon clearly qualifies as a foreseeable outcome of the acts set in motion by the cook. The cook was also involved in an intent-to-cause-serious-bodily-injury homicide. He is an accomplice to that offense by providing payment for the act. When an intent to cause serious bodily injury results in death, the actors are liable for the death.
Just hours after a burglary occurred, police officers interviewed a witness to the crime. An officer showed the witness a number of photographs, including one of a teacher. The witness said that the teacher’s photograph looked like it could have been the burglar, but that he was not completely sure. Based on the witness’s description, a police sketch artist drew a rendering of someone who looked generally like the teacher did in the photograph. The witness said that the rendering “kind of” looked like the burglar, but then again, he was not completely sure. Two police officers went to the teacher’s home and requested that he appear in a lineup. The teacher agreed and accompanied the officers to the police station. The witness, who was seated outside of the lineup room, saw the teacher walk into the police station with the officers. During the lineup, which included the teacher and five other men with similar facial hair and body characteristics as the teacher, the witness identified the teacher as the burglar. The teacher was arrested and indicted. At trial, the teacher’s attorney moved to exclude testimony regarding the witness’s identification of defendant at the lineup.
How should the court rule?
(A) The court should grant the motion, because the teacher was not represented by counsel at the lineup.
(B) The court should grant the motion, because based on the “totality of the circumstances,” the lineup was impermissibly suggestive.
(C) The court should deny the motion, because the lineup was not suggestive.
(D) The court should deny the motion, because the teacher did not have the right to counsel at the lineup.
(B) The court should grant the motion, because based on the “totality of the circumstances,” the lineup was impermissibly suggestive.
A student and his friend were both quarterbacks on the local high school football team, but the student was the starting player while his friend rarely got to play. The friend determined that he would get to play if the student was no longer around, so he decided to have the student killed. The friend knew that the student dined every Thursday night at the local coffee shop. On Wednesday
afternoon, he purchased some rat poison from the hardware store. After determining that one spoonful would be enough rat poison to kill an average person, he approached a classmate, who worked as a waiter at the coffee shop. The friend told the waiter of his desire to kill the student and offered to pay the classmate $10,000 to put a spoonful of rat poison into the student’s beverage when he came in to dine Thursday night. The waiter agreed. When the waiter awoke Thursday morning, he was extremely ill, too sick to go to work. He called his brother, who also worked at the
coffee shop, and explained that he had intended to use the rat poison to kill the student but that he was too sick. The waiter offered to pay his brother $5,000 to carry out the plan. The brother agreed and took the rat poison. Later that day, as the brother was driving to the coffee shop, he saw the student walking down the street. Deciding not to wait until that night, the brother took out a gun he kept in the glove box and shot and killed the student. The brother sped away, but was apprehended several blocks later.
If the friend is prosecuted for the murder of the student, which of the following is incorrect?
(A) He is guilty, because he aided and abetted the brother.
(B) He is not guilty, because the student was killed in a manner unforeseen by the friend.
(C) He is guilty, because he hired the waiter to kill the student.
(D) He is guilty, because he is vicariously liable for the acts of his co-conspirator, the brother.
(B) He is not guilty, because the student was killed in a manner unforeseen by the friend.
Police investigating the unlawful sale of copyrighted videos received a tip that much of the copying was taking place in the offices of a small law firm in the suburbs. Based upon the sworn statement of a paralegal at the firm, stating that the copying operation was taking place in the firm’s mailroom, police officers obtained a warrant to search the premises for evidence of an illegal video piracy operation. In executing the warrant, the police approached the loading dock area of the firm and, without alerting the occupants of the building, kicked open the back door and burst inside with guns drawn. The subsequent search of the law firm’s offices produced a large number of illegally copied videos and the arrest of most of the members of the criminal operation. Before trial, the defendants seek to have the evidence of the search excluded as violating their Fourth Amendment rights.
Must the court exclude the evidence?
A Yes, because the police did not knock and announce their presence before executing the warrant.
B Yes, because the manner in which the warrant was executed was unreasonable under the Fourth Amendment.
C No, because the knock-and-announce rule does not always bar the admission of exclusion of evidence obtained in violation of the rule.
D No, because the police do not need to knock and announce their presence before executing a warrant.
The correct answer is: No, because the knock-and-announce rule does not always bar the admission of exclusion of evidence obtained in violation of the rule.
Discussion of correct answer: In executing search warrants, police must generally knock and announce their presence to the inhabitants of the place to be searched before entering forcibly. The main exception to this rule is the presence of exigent circumstances, especially those in which the police could be harmed or evidence could be lost if the police were to knock and announce their presence. In this case, there was no evidence of exigent circumstances, and the police violated the Fourth Amendment by failing to knock and announce. However, unlike most violations of the Fourth Amendment, the knock-and-announce rule does not automatically trigger the exclusionary rule, but requires instead a broader inquiry that takes into account the social costs of exclusion of the evidence. This answer is therefore correct.
A brother and sister robbed a bank. The brother committed the robbery and the sister drove the getaway car. The sister was unaware that the brother had a gun. The brother held the bank teller at gunpoint, and the bank teller gave the brother all of the cash. The brother harmed no one in the bank. As the brother exited the bank, the teller sounded the alarm. The brother rushed into the car and the sister slammed on the gas. While exiting the bank parking lot, the sister hit a bank customer. The bank customer suffered two broken legs that required minor surgery. Although the surgery was a routine one, the bank customer had a deadly reaction to the anesthesia and died.
Can the brother be convicted of a crime?
A The brother is guilty of homicide because he committed a felony murder.
B The brother is guilty of homicide because the crime was inherently dangerous.
C The brother is not guilty of homicide because of the sister’s intervening actions.
D The brother is not guilty of homicide because of the doctor’s intervening actions.
The correct answer is: The brother is guilty of homicide because he committed a felony murder.
Discussion of correct answer: Felony murder is a killing that is proximately caused during the commission of an inherently dangerous felony. There is vicarious liability for felony murder. All felons are liable for the foreseeable homicides that occur during the commission of a crime. Armed robbery is a felony. Although the brother did not commit the killing, he is vicariously liable because the resulting death was foreseeable. The person was injured while the brother and sister were fleeing the crime. The medical complications that arose as a result of the car accident were foreseeable. Test Tip When an innocent party dies because of something that is related to an action the defendant took during a burglary, arson, robbery, rape or kidnapping (BAARK), you must always do a felony murder analyses to see if one of the few exceptions that relieve the defendant of guilty will apply.
A surgeon performing plastic surgery on a patient realized after the procedure that he mistakenly allowed a ring to fall into the patient’s body. The surgeon notified the patient of the mistake, who suffered serious bodily injuries as a result of the ring. The patient filed criminal charges against the surgeon. The prosecutor who received the case went before a grand jury on the charge of aggravated battery, but the grand jury failed to indict the surgeon on that criminal charge.
Can the prosecutor still pursue indictment on this case?
A No, the patient consented to the aggravated battery.
B No, because double jeopardy has attached.
C Yes, because the prosecutor can present the same case to the grand jury for indictment a second time.
D Yes, because the prosecutor can present different, lesser-included charges to the grand jury for indictment.
The correct answer is: Yes, because the prosecutor can present the same case to the grand jury for indictment a second time.
Discussion of correct answer: The Double Jeopardy Clause is not applicable to this fact pattern. Double jeopardy does not attach when a grand jury fails to indict a defendant or when a grand jury hands down a no bill or no true bill. In such a situation, as is the case here, the defendant may be again indicted for the same offense. Also, given that the prosecutor truly believes that the surgeon has committed an aggravated battery, that prosecutor may go forward on the most correct charge.
A defendant and one of his co-workers were recently fired from their jobs. As they were commiserating, the co-worker thought of a great get-rich-quick scheme to defraud his insurance company. He suggested that the defendant shoot him in the arm in a fake robbery attempt so that he could file a phony insurance claim to recover damages for his injury. In this jurisdiction, it is a felony to file a false insurance claim.
As part of their scheme, the defendant went to the co-worker’s home one evening with a loaded gun. The co-worker told the defendant to shoot him in the arm and then flee from the residence. After being shot, the co-worker planned to call 911 and have an ambulance transport him to the hospital. Following the co-worker’s instruction, the defendant shot the co-worker in the arm. The co-worker then telephoned 911 to report the shooting and requested an ambulance immediately be sent to his home.
Believing that the co-worker only suffered a superficial arm wound, the defendant drove home after the shooting. The bullet, however, severed an artery and the co-worker began to bleed profusely. Before the ambulance arrived, the co-worker lost consciousness from the loss of blood and passed out on the kitchen floor. When the paramedics arrived, they were unable to revive him and he was pronounced dead at the scene.
What is the most serious crime for which the defendant properly could be convicted?
A Felony murder.
B Intent-to-inflict-serious-bodily-injury murder.
C Assault with a deadly weapon.
D Involuntary manslaughter.
The correct answer is: Intent-to-inflict-serious-bodily-injury murder.
Discussion of correct answer: There are four types of murder: (1) intentional killing with premeditation or deliberation; (2) intent to inflict serious bodily injury; (3) felony-murder; and (4) depraved-heart (or reckless killing) murder. The defendant intended to inflict serious bodily injury when he deliberately shot the co-worker in the arm. Therefore this answer choice is correct.
Police investigating the unlawful sale of copyrighted videos received a tip that much of the copying was taking place in the offices of a small law firm in the suburbs. Based upon the sworn statement of a paralegal at the firm, stating that the copying operation was taking place in the firm’s mailroom, police officers obtained a warrant to search the premises for evidence of an illegal video piracy operation. In executing the warrant, the police approached the loading dock area of the firm and, without alerting the occupants of the building, kicked open the back door and burst inside with guns drawn. The subsequent search of the law firm’s offices produced a large number of illegally copied videos and the arrest of most of the members of the criminal operation. Before trial, the defendants seek to have the evidence of the search excluded as violating their Fourth Amendment rights.
Must the court exclude the evidence?
(A) Yes, because the police did not knock and announce their presence before executing the warrant.
(B) Yes, because the manner in which the warrant was executed was unreasonable under the Fourth Amendment.
(C) No, because the knock-and-announce rule does not always bar the admission of exclusion of evidence obtained in violation of the rule.
(D) No, because the police do not need to knock and announce their presence before executing a warrant.
(C) No, because the knock-and-announce rule does not always bar the admission of exclusion of evidence obtained in violation of the rule.
Late one evening, a police department received a telephone call from an unidentified woman who reported hearing loud shrieks and screams from a neighboring home. The caller furnished the police with the address where the disturbance was taking place. When the police arrived at the home, they discovered the homeowner, bludgeoned to death. The murder weapon was a bloodstained baseball bat found near the victim’s body. A subsequent investigation revealed that the homeowner had recently been separated from her husband who had since moved to another city. After questioning several of the victim’s friends and relatives, the police concluded that the husband was a prime suspect in the murder. Thus, two detectives went to question the husband about the killing. When they arrived at his apartment, the detectives persuaded the landlord to let them into his apartment. Finding no one inside, they searched the apartment. Before leaving, the detectives took a box of cereal, which they planned to use for a fingerprint comparison. The prints from the cereal box matched those found on the baseball bat. The police presented the fingerprint comparison to the grand jury investigating the murder. The husband is to testify before the grand jury. Before his appearance, the husband files a motion to evidence of the fingerprint comparison, contending that the evidence was illegally obtained.
How should the court rule on his motion?
(A) Granted, because the warrantless search of the husband’s apartment was unconstitutional.
(B) Granted, because the grand jury is barred from considering illegally obtained evidence.
(C) Denied, because the exclusionary rule has not been extended to grand jury hearings.
(D) Denied, because the landlord had the apparent authority to authorize the search of the husband’s apartment.
Denied, because the exclusionary rule has not been extended to grand jury hearings.
A sculptor had been in a romantic relationship with a man, and the man often sat as a model for the sculptor who created various sculptures based on the man’s physique. The man came to find that the sculptor had been carrying on an affair with a fellow artist, and that the sculptor had not really been interested in a long-term relationship but had primarily been interested in him as an artistic subject. The man asked the sculptor to end the relationship with the artist, but the sculptor refused and instead broke off the relationship with the man. A month later, the man saw an article in a local paper that included pictures from the sculptor’s art studio. One picture featured a life-sized sculpture of the man that she had previously promised to give to the man, and the caption indicated it was to be sold to a collector for $50,000. Furious, the man broke into the sculptor’s art studio and set fire to the statue. Soon the fire spread and one entire room of the three-room art studio was severely damaged by fire.
If the man is charged with common law arson, what is his best defense?
A He intended to burn only the sculpture, and did not intend to damage the studio itself.
B The sculptor had promised to give the statue to the man in return for $5,000 he provided to her to buy the sculpture materials.
C The sculptor conducted work at the studio, but neither she nor anyone else lived there.
D Only one portion of the studio burned, while the majority of the studio was undamaged.
The correct answer is: The sculptor conducted work at the studio, but neither she nor anyone else lived there.
Discussion of correct answer: At common law, arson consisted of the malicious burning of the dwelling of another. While a majority of states have expanded the common law requirement of a dwelling to include any type of structure, under common law, the man would not be guilty of arson because no one lived in the art studio, and thus it was not a dwelling.
A man’s mother has lived with him for 20 years, and for the past several months she has been suffering from terminal cancer. Tired of her constant demands, the man begins to add a small amount of poison to his mother’s liquid medication each day. When the mother dies, the medical examiner determines that the cancer would have killed her within a few days, but that her death was hastened by the poison.
If the man is prosecuted for murder, what should the result be?
A The man should be convicted, because his actions showed a depraved indifference to the value of human life.
B The man should be acquitted, because his mother was ill and in pain.
C The man should be acquitted, because his mother would have died almost immediately from cancer.
D The man should be acquitted, because the man’s actions alone would not have caused the death.
The correct answer is: The man should be convicted, because his actions showed a depraved indifference to the value of human life.
Discussion of correct answer: The act of introducing a poison into the mother’s medication does show a reckless indifference to the value of her life. While the facts do make it clear that the mother would have died very soon anyway, the son’s actions hastened that death and were done in a way that demonstrated an indifference to whatever life his mother still had left. Because these actions will meet the standard for malice, he will be convicted of her murder.
While on patrol, an officer stopped a man for driving while intoxicated. The officer arrested the man and immediately transported him to a local hospital for a blood sample to test his blood-alcohol content. At the hospital, the officer asked the man for his consent to the blood draw, but the man refused. The officer directed the lab technician to take the blood sample anyway. The man’s blood-alcohol level proved to be twice the legal limit. At trial, the man claimed the blood draw was unlawful because the officer failed to obtain a warrant. The officer responded that although he had quickly received blood draw warrants in the past, he believed he was under no duty to do so.
Was the blood draw unlawful?
A No, because the blood-alcohol of a drunk-driving suspect will diminish over time.
B No, because the procedure for obtaining the blood sample was reasonable.
C Yes, because the facts did not show the evidence was likely to disappear before a warrant was obtained.
D Yes, because the officer conducted the blood draw without the man’s consent.
The correct answer is: Yes, because the facts did not show the evidence was likely to disappear before a warrant was obtained.
Discussion of correct answer: In certain emergency situations where evidence may be lost or destroyed before a warrant can be obtained, a warrantless search and seizure is permitted [Illinois v. McArthur, 531 U.S. 326 (2001); Schmerber v. California, 384 U.S. 757 (1966)]. This is known as exigent circumstances. Importantly, the police may conduct a warrantless search and seizure of evidence in or on a suspect’s body provided that: (1) there is probable cause to believe that the nature of the evidence renders it easily destroyed or likely to disappear before a warrant can be obtained; and (2) the procedure for seizing the evidence is reasonable. While blood-alcohol level is a classic example where exigent circumstances usually apply, the problem here is that the officer stated that he could have quickly obtained a search warrant but chose not to do so. There was no indication the evidence was likely to disappear before a warrant could be obtained. Such a situation fails the first part of the test for exigent circumstances [Missouri v. McNeely, 569 U.S. 141 (2013)].
A legislature, determined to do something about impatient drivers who put others at risk, enacts a law that driving in the breakdown lane without a legitimate reason to do so is a misdemeanor. The same act, if it proximately causes serious injury, is a felony. Several months later, the defendant was stuck in traffic and tried to go around the traffic bottleneck by driving in the breakdown lane. After driving in the breakdown lane for about a quarter of a mile, the defendant’s vehicle crested a hill. Just past the crest, he saw a stalled car in the breakdown lane. The defendant could not stop in time, so he tried to swerve back into the right-hand lane of the highway. His car struck a minivan in the right-hand lane, which flipped over, killing the driver.
What is the most serious crime of which the defendant can be convicted?
A Involuntary manslaughter.
B Depraved-heart murder.
C Intent-to-cause-serious-bodily-injury murder.
D Felony murder.
The correct answer is: Involuntary manslaughter.
Discussion of correct answer: Involuntary manslaughter, sometimes referred to as “misdemeanor manslaughter,” is an unintentional killing resulting without malice aforethought caused either by gross or criminal negligence (higher than that required for tort liability) or during the commission or attempted commission of an unlawful act. In this case, the unintentional killing of the minivan driver occurred during the defendant’s unlawfully driving in the breakdown lane. Additionally, the dangerous operation of a motor vehicle is often considered criminal negligence. As such, through either the misdemeanor-manslaughter rule or because the minivan driver’s act constituted criminal negligence, the most serious crime for which the defendant could be convicted would be involuntary manslaughter. Therefore, this answer is correct.
A defendant wished to see his high school basketball team win the state championship. During an important game, the defendant pulled out a gun and shot at the leg of a key player on the opposing team. The defendant intended only to inflict a slight wound so that the opposing player would be unable to complete the game. When the defendant fired the shot, he unintentionally hit a player on his own high school team in the chest, killing him instantly.
What is the most serious crime of which the defendant can be convicted?
A Murder.
B Voluntary manslaughter.
C Involuntary manslaughter.
D Battery.
The correct answer is: Murder.
Discussion of correct answer: At common law, murder is the unlawful killing of another human being with malice aforethought. Malice is found where there was intent to kill, intent to inflict serious bodily harm, reckless indifference to human life, or a death caused during the commission of a dangerous felony. Although the defendant may have intended only to cause a slight wound, his actions did, in fact, cause the death of another human being. Such a killing is murder unless justified, excused, or mitigated. The fact that the defendant shot the “wrong” basketball player would not change the outcome, because he was aware (or should have been aware) that the shooting created a serious risk of death or grave bodily harm. Therefore, the malice is present, and the defendant can be convicted of murder, since there is no mitigation, justification, or excuse.
A police department received reports of violent attacks occurring in the afternoon, just after the local high school let out for the day. The police put together a task force to gather intelligence concerning students who might have gang affiliations. Once the police compiled this list of gang-associated students, they asked the principal to search the listed students, and to contact the police if they found any evidence. Of the 25 students searched, only one was found to be carrying a weapon. In the student’s ensuing trial, his lawyer moved to suppress the weapon.
Is the court likely to grant the student’s motion?
A No, because the school did not need a warrant to search a student.
B No, because the student had no reasonable expectation of privacy in school.
C Yes, because the police did not conduct the search.
D Yes, because there was neither probable cause nor a search warrant in support of the search.
The correct answer is: Yes, because there was neither probable cause nor a search warrant in support of the search.
Discussion of correct answer: The Fourth Amendment generally protects only against governmental conduct and not against searches by private persons–it does not prevent the introduction of evidence obtained by illegal searches done by private citizens [Burdeau v. McDowell, 256 U.S. 465 (1921)]. However, governmental “agents” include citizens acting under police direction. Here, the school officials were acting on behalf of the police. They would, therefore, be held to the same standard. Without probable cause and a search warrant (or warrant exception), the evidence will be considered illegally seized and will be inadmissible against the student. Here, there was neither probable cause (the only fact in support of the search was that the student was “gang associated”) nor a warrant or applicable exception.
A man owned a home security business. In addition to selling personal defense devices, he also trained dogs to attack strangers at night. He sold and leased his guard dogs to various business and factory owners, who used the guard dogs to frighten away intruders from entering their premises at night. One evening, the man was in the backyard of his home training three of his guard dogs. The backyard was enclosed with a chain-link fence and a latched gate that prevented the dogs from running out. After the training session, the man opened the gate and permitted the dogs to run loose in his front yard. Minutes later, a bicyclist got a flat time in from of the man’s home and was attacked by several of the dogs. The bicyclist suffered severe injuries and died as a result of the attack. At trial, it was conceded that the man’s dogs attacked a postal worker two weeks prior to this incident.
For what crime should the man should be found guilty?
A Murder, because the man’s actions were malicious.
B Involuntary manslaughter, because the man did not command the dogs to attack the bicyclist.
C Voluntary manslaughter, because the dogs could have reasonably believed the bicyclist was an intruder.
D Reckless endangerment, because the dogs killing the victim was a superseding cause.
The correct answer is: Murder, because the man’s actions were malicious.
Discussion of correct answer: Depraved-heart murder occurs when a defendant’s conduct creates an unjustifiable, very high degree of risk of death or serious bodily injury to another, even though unaccompanied by any intent to kill or to do serious bodily injury, and such conduct actually causes the death of another. Because the man had trained his guard dogs to attack at night and then opened the gate to let them run loose, he will be criminally responsible for the death of the bicyclist. This is so especially in light of the fact that he was aware that these dogs had previously attacked postman postal worker just two weeks prior to this incident.
A property owner’s old bulldog didn’t like the family’s new puppy, and would hide from the puppy during the day so as not to have his naps interrupted by an impromptu game of “chew old bulldog’s ear.” One warm evening, the old bulldog hid in the tool shed, and was so deeply asleep that he didn’t awaken when his owner shut and locked the shed for the night. When he awoke about midnight and discovered that he was locked in, the old dog began jumping against the shed door. The loud banging of the tool shed door aroused the property owner from sleep. Thinking that his house was being burglarized, the property owner grabbed his loaded shotgun and tiptoed out onto the back patio from where he had heard the loud banging. In the dark of the patio, the property owner did not notice that his young daughter had left her jacks scattered about on the patio. When his bare feet encountered the jacks, the property owner was soon hopping in pain and fell over a patio table, crashing heavily to the floor. Upon impact, the shotgun went off, both barrels blasting a hole through the patio screen. The pellets struck the property owner’s neighbor, who had also heard the thumping in the tool shed and had climbed over the fence which separated their yards. The neighbor was killed instantly. Which of the following is the property owner’s strongest defense to a charge of involuntary manslaughter as a result of the neighbor’s death?
The property owner’s actions that night did not constitute criminal negligence.
Believing a woman had been involved in a drive-by shooting, police officers went to her grandmother’s apartment to search for the weapon. The grandmother told the officers that she owned the apartment and had the only keys, but that her granddaughter had been living in the first spare bedroom for several months. The grandmother gave police permission to search the entire apartment, and the officers discovered a handgun in the second spare bedroom. The grandmother said the second spare bedroom was kept open and that she allowed her other grandchildren to sleep there whenever they desired, which was quite often. The police soon connected the handgun to the shooting. At trial, the woman asserted that the search violated her Fourth Amendment rights.
Was the search of the second bedroom lawful?
A No, because the police did not have a warrant.
B No, because the woman had a reasonable expectation of privacy in the apartment.
C Yes, because the grandmother owned the apartment.
D Yes, because the other grandchildren used the second bedroom.
The correct answer is: Yes, because the other grandchildren used the second bedroom.
Discussion of correct answer: A person may assert Fourth Amendment rights only when he or she had a reasonable expectation of privacy in the premises searched. While an overnight guest in another’s home has standing, that privacy interest only extends to those areas where the guest can reasonably expect privacy. If a person with either actual or apparent authority gives consent to search, a search within the scope of that consent may be conducted without a warrant. Here, while the woman slept in the first bedroom in the apartment, it was clear that her grandmother owned the apartment, did not intend to give her granddaughter unfettered access to it, and intended the second bedroom to be shared by her other grandchildren. As such, the woman had no legitimate expectation of privacy in the second bedroom where the gun was found. Consequently, the police could rely on the grandmother’s consent to search the entire apartment when searching the second bedroom.
A senior associate in a law firm was conducting spot checks of other associates as they left the office to determine who had taken some documents that were missing from the firm’s library. A young associate was leaving his office for the day when he was stopped by the defendant, who asked him to open his briefcase. The senior associate explained that some files from the law office had been stolen. Startled, the young associate refused to permit the senior associate to look in his briefcase. The young associate said that he had personal papers in his briefcase and he didn’t want the senior associate to see them. Having his suspicions aroused, the senior associate raised his clenched fist and exclaimed, “If you don’t hand over that briefcase, I’m going to punch you in the mouth.” The young associate, who, unknown to the senior associate, suffered from a serious heart condition, had a seizure and died as a result of fright produced by the threatened attack.
Which of the following is the most serious crime for which the senior associate should be found guilty?
A Assault.
B Voluntary manslaughter.
C Involuntary manslaughter.
D Murder.
The correct answer is: Involuntary manslaughter.
Discussion of correct answer: As a general rule, whenever an intentional battery or assault results in an unintended death, the defendant is guilty of involuntary manslaughter. LaFave notes that a criminal assault, like a criminal battery, is an unlawful act malum in se. Therefore, if the defendant approaches close to another person intending to strike him but not to kill him, and the latter, who, unknown to the defendant, possesses a weak heart, has a heart seizure and dies as a result of fright produced by the threatened attack, the defendant is guilty of manslaughter, though he never touched the victim. See Regina v. Dugal [4 Que. L.R. 350 (Q.B. 1878)], referred to by LaFave in Criminal Law, pg. 601.
A man’s mother has lived with him for 20 years, and for the past several months she has been suffering from terminal cancer. Tired of her constant demands, the man begins to add a small amount of poison to his mother’s liquid medication each day. When the mother dies, the medical examiner determines that the cancer would have killed her within a few days, but that her death was hastened by the poison.
If the man is prosecuted for murder, what should the result be?
a. The man should be convicted, because his actions showed a depraved indifference to the value of human life.
b. The man should be acquitted, because his mother was ill and in pain.
c. The man should be acquitted, because his mother would have died almost immediately from cancer.
d. The man should be acquitted, because the man’s actions alone would not have caused the death.
a. The man should be convicted, because his actions showed a depraved indifference to the value of human life.
A defendant was walking through a park when he decided to rob someone. He hid behind a tree, lying in wait for a victim to approach. Shortly thereafter, a girl was strolling in the park when the defendant suddenly jumped from his hiding place and accosted her. Although the defendant intended only to rob his victim, he punched her in the mouth and she fell to the ground. The defendant then grabbed her purse and fled. Unknown to the defendant, the girl suffered a fractured skull when her head struck the pavement.
She subsequently died from her head injuries.
Which of the following is the most serious crime for which the defendant can be found guilty? (A) Murder. (B) Felony murder. (C) Involuntary manslaughter. (D) Voluntary manslaughter.
(B) Felony murder.
A coach of a high school football team took his team all the way to the state championship game. Although the team lost, the owner of the local professional football team was so inspired that he donated his own team’s championship trophy to the high school for one year, and the high school’s board gave the coach discretion to use the trophy as he saw fit. Shortly thereafter, a wealthy football enthusiast contacted the coach and offered to make it “worth his while” to have the trophy shipped to him and even offered to have a replica made, promising that no one would ever know the difference. The coach accepted the offer and sent the real trophy to the wealthy football enthusiast, and put the replica trophy in the high school’s display case. The owner visited the high school to see the display and promptly noticed that the trophy on display was a fake. The wealthy football enthusiast disappeared with the real trophy and never paid the coach.
With what crime may the coach be properly convicted?
A Larceny.
B Larceny by trick.
C False pretenses.
D Embezzlement.
The correct answer is: Embezzlement.
Discussion of correct answer: Embezzlement is the fraudulent conversion or misappropriation of the property of another by one who is already in lawful possession. Here, the coach had lawful possession of the trophy but only for purposes of displaying it at the high school for a year. The owner retained ownership of the trophy despite the coach’s possession. When the coach sent the trophy to the wealthy sports enthusiast, he made an unlawful conversion seriously interfering with the owner’s rights. It is irrelevant that the coach was never paid for the trophy.
During the investigation of several serial killings, a concerned citizen claiming to have information about the killer reported to police on the hotline. The officers visited the citizen in his home who began to give the officers details about the crimes that only the killer would know. The police, suspecting that the citizen might be the killer, decided not to give the citizen his Miranda warnings and asked him if he would give them a taped interview at the police station. The citizen agreed to meet the police the next day at the police station to give the interview. After the interview was completed, the police arrested the citizen and charged him with multiple murders. At trial the defense filed a motion to suppress the statements because the police had not given the defendant his Miranda warnings before giving the taped statement.
Should the motion be granted?
(A) No, because the citizen was free to leave and volunteered to give the statement.
(B) No, because the citizen originally contacted the police on the hotline to provide information and the statement was voluntary.
(C) Yes, because the police had made the subjective determination not to read the citizen his Miranda warnings.
(D) Yes, because the citizen believed that he was not free to leave.
(A) No, because the citizen was free to leave and volunteered to give the statement.
A woman was an avid kayaker and often spent her free time kayaking in the river near her home. One day, a man brought a kayak with him to the river to try it out and, seeing the woman, asked her if she would help him. The woman recognized him as having once taken her out on a date, and he had treated her very poorly and now did not even seem to remember her. The woman told him that she did not know enough about kayaking to help him, although she knew that the river was very choppy that day and she could see that the man’s kayak was in poor condition. The man went out in his kayak, but it was quickly damaged and began to sink. He could not free himself from the kayak. He yelled at the woman asking for her help, but she just laughed in return. The man drowned.
Which is the most serious crime for which the woman can be convicted?
A Murder.
B Voluntary manslaughter.
C Involuntary manslaughter.
D No crime.
The correct answer is: No crime.
Discussion of correct answer: As in tort situations of nonfeasance, people who fail to prevent injury or death are generally not criminally liable for the victim’s condition unless they have a duty to act. Here, the woman had no duty to warn or help the man, so she is not guilty of homicide. As a test tip, here, you may think she is a bad person, but she has no duty to help, as she did not cause his peril, thus is not guilty of any crime.
A longtime Bostonian was a huge Red Sox fan. Following a nighttime game, she picked up the red satin Red Sox jacket on the seat next to her and walked up the stadium steps toward the exit. Suddenly, she was accosted by another woman, who grabbed the fan’s arm and accused her of stealing her jacket. The other woman grabbed the jacket, which had been slung over the fan’s arm, and clutched it tightly. The fan was appalled and tried to grab the jacket back, simultaneously kicking the other woman in the shin. The other woman lost her balance and began to fall. Meanwhile, a bystander who believed that the fan was attempting to steal the jacket from the other woman, rushed to the other woman’s aid, but the woman’s flailing limbs caused the bystander to lose her balance as well, and they both tumbled down the steps of the stadium. When the Red Sox fan saw them both lying motionless on the bottom landing, she decided that her jacket wasn’t worth all this trouble. She hastily exited the ballpark. The other woman gradually regained consciousness and stood up. She shakily made her way to her car, but before she could unlock the door, she blacked out again and fell, and was hit by a car driven by a man who had been talking on his mobile phone while driving without using an earpiece, in violation of state statute. Realizing that he had hit someone, the man stopped and called an ambulance for the woman, but it was too late. The woman died en route to the hospital. As it turned out, the jacket did belong to the other woman. She had been sitting two seats away from the Red Sox fan and had placed the jacket on the seat between them. The fan had mistaken it for her own identical jacket, which she later found in her backpack. What is the most serious crime of which the man who drove the car can be convicted?
(C) Involuntary manslaughter.
One night, two teenagers, ages 17 and 18, were walking home from their jobs at a pizza shop. As they passed a large house, the older boy told the younger one that as a kid, he had worked in a hotel for the family that owned both that house and the hotel in which he worked. He also told the younger boy that when he had worked at the hotel, rumor had it that the hotel owner skimmed most of the cash off the top of his businesses, and that he had accumulated more than $500,000 in cash, which he kept hidden in the basement of his home. The younger boy grew extremely excited at the thought of all that cash there for the taking. They spontaneously decided to sneak into the house and steal the money. They jumped the fence surrounding the property and crept through the backyard of the house. They were attempting to jimmy the downstairs window when they heard the owner yelling, “Hey, who’s there?” Frightened, the boys began running away, chased by the incensed homeowner. However, before he could catch up to them, the homeowner suffered a heart attack from the excitement and extreme physical exertion and died.
If the teenagers are charged with homicide, they will likely be found guilty of what crime?
A Felony murder.
B Involuntary manslaughter.
C First-degree murder.
D No crime, because they intended no death.
The correct answer is: Felony murder.
Discussion of correct answer: Felony murder is an intentional or accidental killing proximately caused during the commission or attempted commission of a serious or inherently dangerous felony, including burglary, arson, robbery, rape, and kidnapping (known through the mnemonic “BARRK”). The resulting death must be a foreseeable consequence of the defendant’s actions. Courts have generally been very liberal in applying the foreseeability requirement, and for purposes of felony murder most deaths are considered foreseeable. At modern law, the crime of burglary consists of the breaking and entering of the structure of another with the intent to commit a felony or theft offense therein. Here, as the teenagers jumped the fence of the homeowner’s property and attempted to jimmy the window open with the intent to commit larceny therein, they committed attempted burglary. Because it is foreseeable that a burglary victim may chase after the burglars, it is likely that the homeowner’s death would be considered a foreseeable outcome of the teenagers’ act. As such, both teenagers will likely be found guilty of felony murder.
A husband returned home early from a business trip and found his wife in bed in the midst of sexual intercourse with his best friend. A verbal disagreement ensued and the husband’s best friend left. One week later, the husband showed up at the best friend’s house and shot and killed him. The police arrived at the best friend’s house and arrested the husband. This jurisdiction does not have a murder statute but follows the common law.
Which crime is the husband likely guilty of committing?
A Voluntary manslaughter, because seeing his wife with his best friend was adequate provocation for the homicide.
B Voluntary manslaughter, because the husband lacked the requisite malice to commit murder.
C Murder, because the husband had enough time to cool off after finding his wife in bed with his best friend before committing the homicide.
D Murder, because seeing his wife with his best friend was not adequate provocation for the homicide.
The correct answer is: Murder, because the husband had enough time to cool off after finding his wife in bed with his best friend before committing the homicide.
Discussion of correct answer: While finding his wife in bed with his best friend constituted adequate provocation to reduce the husband’s criminal liability from murder to voluntary manslaughter, because the husband waited one week before killing his best friend, he was no longer in the heat of passion at the time he committed the homicide, and therefore, the homicide cannot be mitigated to voluntary manslaughter and the husband is criminally liable for the murder of his best friend.
A defendant had an argument with a patron in a bar over a game of pool. As they were quarreling, the defendant pulled out his knife, intending only to frighten the patron. The knife was six inches long, and the defendant held it out towards the patron. The patron immediately retreated from the pool table, telling the defendant that he was a hemophiliac and that he did not want any trouble. Wanting to have some fun at the patron’s expense, the defendant drew near and swung the knife in the patron’s direction. The defendant got a little too close to the patron and unintentionally nicked the patron’s arm. The bar was far from a hospital, and by the time the patron arrived at the emergency room, he had died from his injury.
What is the most serious crime for which the defendant could be convicted?
A Murder.
B Voluntary manslaughter.
C Involuntary manslaughter.
D Battery.
The correct answer is: Murder.
Discussion of correct answer: While the defendant only intended to frighten the patron (which would be an assault) and did not intend to kill him or to inflict serious bodily harm, there is malice, because he was aware that the knife could cut the patron, who was a hemophiliac. As such, the defendant could be convicted of murder.
A department store had experienced a growing incidence of shoplifting. At the store’s request, the police concealed a woman, who was a detective, at a vantage point above the women’s apparel fitting rooms, where she could see into these rooms as customers tried on clothes. The detective saw the defendant enter a fitting room, stuff a dress into her pocketbook, leave the fitting room, and start for the street door. By prearranged signal, the detective notified another police officer near the door, who detained the defendant as she started to go out into the street. The defendant was placed under arrest and the dress was retrieved from her purse. The defendant was subsequently charged with shoplifting. At trial, the defendant moved to prevent the introduction of the dress into evidence.
Will her motion be granted?
A Yes, because the police should have secured a search warrant to search her bag.
B Yes, because a customer has a reasonable expectation of privacy while using a department store fitting room.
C No, because the search and seizure were made incident to a valid arrest based on probable cause.
D No, because the detective could see into the room, and thus, the defendant’s activities were legitimately in plain view.
The correct answer is: Yes, because a customer has a reasonable expectation of privacy while using a department store fitting room.
Discussion of correct answer: The Fourth Amendment prohibits unreasonable searches and seizures by the government without probable cause in places where an individual would have a reasonable expectation of privacy. The police detective did not have the right to look into the dressing room as the defendant was changing solely on the basis that there had been an increase in shoplifting at the store. The fact that the store invited the police to look in the dressing room does not excuse the police of the requirements of the Fourth Amendment.
A tenant in an apartment building regularly stole decorative items from the lobby of his building. The landlord suspected the tenant of stealing from him, and asked a police officer friend to look into this. The officer came to the tenant’s door, and told him that she had just moved in but was locked out, and asked whether she could stay in the tenant’s apartment while she waited for a locksmith. The tenant allowed her to come inside. While inside, the police officer looked around and noticed that all the decorative items the landlord had mentioned were prominently on display in the tenant’s apartment. The tenant noticed the officer looking at the items and told her she had to leave immediately. The officer returned with a warrant the following week and seized the items. The tenant was arrested for larceny. At trial, the tenant filed a motion to suppress the introduction of the seized items.
Will this motion be successful?
A Yes, because the items were the fruit of an illegal search.
B Yes, because the tenant revoked his consent to the search.
C No, because the tenant only revoked consent after the officer observed the stolen items in plain view.
D No, because consent given to an invalid warrant is deemed involuntary.
The correct answer is: Yes, because the items were the fruit of an illegal search.
Discussion of correct answer: Although consent is an exception to the warrant requirement, consent is not given when a police officer enters a residence by fraud, i.e., hiding his or her identity as an officer. Here, the officer entered the tenant’s residence by fraud. Although she later obtained a warrant, this was based on an illegal search, making the seizure the fruit of the poisonous tree and therefore inadmissible. As a test tip, evidence will be suppressed when it is the result of police misconduct. In this case, it is unfair for the police to gain consent to enter an apartment by lying to the occupant.
An usher at a grand old movie theater learned that the theater’s manager had recently inherited a rare print of the usher’s favorite movie. The usher asked the manager if he could borrow it. The manager declined, but said he would bring the print into the theater so the usher could see it. The usher had been reading up on the subject of ESP quite a bit, and mistakenly believed there was an extremely high likelihood that he could move large objects with his mind. When the manager brought the print into the theater, the usher pointed up to the enormous chandelier above the manager’s head and said that if the manager did not give him the print, he would use the power of his mind to make the chandelier fall on the manager’s head. The manager looked up at the massive chandelier, laughed and told the usher he was insane to think he could move the chandelier with his mind. Nevertheless, the manager handed the print over to the usher, who was relieved, as he never had any intention of using his self-perceived powers of ESP to hurt the manager. While the usher had intended only to keep the print temporarily and then return it to the manager, he enjoyed it so much that decided to keep it permanently. The usher was charged with robbery.
What is the defendant’s best defense to the charge of robbery?
A The manager did not fear that the chandelier would fall on him.
B The usher did not have the ability to make the chandelier fall on the manager.
C The usher never intended to use force against the manager.
D The usher did not intend to deprive the manager of the print permanently at the time of the taking.
The correct answer is: The manager did not fear that the chandelier would fall on him.
Discussion of correct answer: Robbery consists of all the elements of larceny, plus two additional elements: (1) the taking must be from the person or in the presence of the person; and (2) the taking must be accomplished by force or violence or by intimidation of the threat of violence. If the robbery is by threat of violence, the victim must be in actual fear at the time of the taking. Here, the manager indicated he was not in fear of the usher using ESP to hurt him. Thus, while the usher may well be guilty of larceny, he would not be guilty of robbery.
A violinist was planning a vacation to Europe and was making final arrangements for the three weeks she would be out of the country. The last thing she needed to do was to make arrangements for her prize golden retriever dog. The violinist asked her neighbor if he would be willing to keep the dog and care for him until she returned from Europe. The violinist said that she would pay for all the food costs for the dog and give the neighbor an additional $500 for his efforts. The neighbor agreed to care for the dog until the violinist returned from Europe.
Two weeks later the neighbor was walking the dog when he met a breeder, who ran a business breeding dogs and selling their offspring. The breeder saw the dog and recognized that the dog was a purebred, worth at least $5,000. The breeder offered to buy the dog on the spot for $2,500. The neighbor quickly accepted the money and handed over the dog to the breeder.
When the violinist returned from Europe, the neighbor falsely told her that the dog had run away. He then returned the money the violinist had given to him before she left.
In a jurisdiction that recognizes larceny, larceny by trick, embezzlement, and false pretenses as separate crimes, of which is the neighbor guilty?
A Larceny.
B Embezzlement.
C Obtaining property by false pretenses.
D Larceny by trick.
The correct answer is: Embezzlement.
Discussion of correct answer: Embezzlement is the fraudulent conversion of property by one in rightful possession of the property. Under these circumstances, the neighbor would be guilty of embezzlement because the violinist entrusted the dog to the neighbor and then the neighbor caused the violinist to be permanently deprived of the dog as a result of his sale to the breeder.
A man is just over 4’10’’ tall and weighs exactly 100 lbs. His small size has always left him feeling powerless, and he decides to overcome that feeling of helplessness by robbing the local supermarket. He shares his plan with his best friend. The friend initially tries to change the man’s mind, but when he is unable, he agrees to act as the driver, because he is concerned about the man’s safety. He pulls right up to the door of the supermarket, retrieves a tire iron from the trunk, and hands it to the man. As the man enters the store with the tire iron in hand, the friend has second thoughts and drives away. The man approaches the manager, a former middle linebacker for his university team, brandishes the tire iron, and tells him to empty the register. The customers waiting in line laugh and the manager mockingly asks what exactly the man is planning to do with that tire iron. With that, the man panics, grabs a dispenser of lottery tickets off the counter, and runs. He is quickly apprehended.If the man and the friend are charged with robbery, what should be the outcome?
Neither the man nor the friend should be convicted.
A husband received a telephone call from an unidentified source who informed him that his wife was having an affair with his friend. Enraged, the husband drove to his friend’s house with a loaded shotgun. Believing that his friend was in an upstairs bedroom, the husband fired the shotgun through the first-floor dining room window to teach him a lesson. Unbeknownst to the husband, his friend’s wife was in the dining room and suffered a minor gunshot wound. Of which crime(s) can the husband be convicted?
Neither attempted murder of his friend, nor attempted murder of his friend’s wife.
A defendant was arrested for beating his girlfriend. In the patrol car, before he was Mirandized, the defendant spontaneously told the police officer that he had been drunk and did not know what happened. The defendant was then Mirandized and he immediately invoked his right to remain silent. At trial, the prosecutor asked the police officer what the defendant had told him before the defendant was Mirandized. The defendant’s attorney objected.
How will the court rule on the objection?
(A) Sustained, because it is not proper to comment on the defendant’s invocation of his right to remain silent.
(B) Sustained, because his statement was obtained after he was placed in handcuffs.
(C) Overruled, because the defendant made a voluntary statement which can be used to impeach him at trial.
(D) Overruled, because the defendant’s statement was made prior to a Miranda warning and he was not being
interrogated when he made the statement.
(D) Overruled, because the defendant’s statement was made prior to a Miranda warning and he was not being
interrogated when he made the statement.
A police officer observed an executive leaving an office building smoking what appeared to be a marijuana cigarette. The police officer approached the executive and identified the smell of marijuana. The officer arrested the executive on the street for marijuana possession. The officer then searched the executive’s suit jacket and found a sealed envelope which contained a white powder later testing positive for cocaine. The officer also ordered the defendant to remove his shoes. Another envelope was seized which also contained a white powder later testing positive for cocaine. The executive did not consent to the search nor did the officer have consent to open the two sealed envelopes. At trial on charges of drug possession, the executive filed a motion to suppress the cocaine seized after the arrest.
Should the court grant the motion?
A Yes, because the stop and frisk went beyond the scope of a legitimate pat down for weapons.
B Yes, because the officer needed a warrant to open the envelopes.
C No, because the search was incident to a lawful arrest.
D No, because the officer had probable cause to conduct the search after observing the executive smoking marijuana.
The correct answer is: No, because the search was incident to a lawful arrest.
Discussion of correct answer: Where police make a lawful arrest, the police may search the defendant’s person incident to the arrest to protect the safety of the officers and retrieve any evidence or contraband.
A breeder and owner of vicious guard dogs trained his dogs to attack strangers at night. He often sold and leased his guard dogs to various business and factory owners who used the guard dogs to frighten away intruders from entering their premises at night. One evening, the breeder was in the back yard of his home training three of his guard dogs. The back yard was enclosed with a chain link fence and a latched gate that prevented the dogs from running out. After the training session, the breeder opened the gate and permitted the dogs to run loose in his front yard.
Minutes later, a man was walking along the sidewalk In front of the breeder’s house when he was attacked by one of the dogs. The man suffered severe injuries and died as a result of the attack. The breeder should be found guilty of what crime? (A) Murder. (B) Involuntary manslaughter. (C) Voluntary manslaughter. (D) Reckless endangerment.
(A) Murder.
A defendant shot and killed a victim at close range with a revolver. After the shooting death, the defendant was arrested and charged with first-degree murder. In this jurisdiction, first-degree murder is defined as “knowingly or willfully causing the death of another human being.”
Which of the following situations, if believed by the jury, would most likely result in the defendant’s acquittal of the first-degree murder charge?
(A) The victim, who was the defendant’s brother, suffered from an incurable case of cancer and asked the defendant to kill him to put an end to his pain and suffering. (B) The killing was not done with premeditation or deliberation. (C) The defendant intended to kill himself, but the bullet grazed his head and struck and killed the victim. (D) The defendant intended to kill another man, but the victim unknowingly stepped in front of the other man and was struck and killed by the bullet.
(C) The defendant intended to kill himself, but the bullet grazed his head and struck and killed the victim.
An accused mobster had posted bail and was home, awaiting the start of his trial. Unbeknownst to him, one of his former associates had agreed to work for police in exchange for immunity on charges filed against the associate. The associate, at the direction of the police, brought the mobster a box of fancy cigars to celebrate the mobster’s homecoming. The two sat down to a game of poker and light the cigars. The associate started joking with the mobster about the upcoming trial and talked about specific details of the crimes for which the mobster was accused. The associate asked the mobster if the associate should do anything to help conceal some of the mobster’s crimes and the mobster gave the associate specific instructions on what to do, thereby implicating the mobster in every crime for which the mobster was accused.
Which of the mobster’s constitutional rights were violated during the exchange?
(A) The mobster’s Fifth Amendment right to counsel.
(B) The mobster’s Fourth Amendment right against illegal searches.
(C) The mobster’s Fifth Amendment right requiring Miranda warnings.
(D) The mobster’s Sixth Amendment right to counsel.
(D) The mobster’s Sixth Amendment right to counsel.
A widow, desperate for money, decided to defraud her insurance company. She convinced a high school friend to steal her diamond tennis bracelet and earrings worth thousands of dollars. They would then split the insurance proceeds. She would leave her bathroom window open with the expensive jewelry on the table just under the window. The high school friend would reach in and steal the jewelry. The widow would then report the loss to her insurance company, and she would split the proceeds with the friend. Thinking it sounded like easy money, the friend agreed. The friend told the widow that he would take the jewelry the next Friday night. On the designated Friday night, the widow carefully set the jewelry on the table and left the window open before she went to bed. Meanwhile, a local drug addict was in the neighborhood looking for an easy target. He noticed the open window at the widow’s house and looked inside to see if there was anything inside worth stealing. He noticed the jewelry, pushed the window in slightly to reach the jewelry, and stole it. He was later apprehended and charged with burglary. The jurisdiction follows the modern trend in its definition of burglary.What is the likely outcome of the burglary charge against the drug addict who stole the widow’s jewelry?
Guilty, because he met the modern requirements for burglary.
An off-duty police officer came home early from work one day. He walked up to his bedroom and, upon opening the door, saw his wife and another man in bed together. Visibly upset, the husband grabbed his service revolver and shot the man to death. He then went to a local bar, had several drinks, returned home, and killed his wife.
The husband is guilty for which of the following crimes?
A Murder of both his wife and the other man.
B Voluntary manslaughter of both his wife and the other man.
C Murder of the other man, and voluntary manslaughter of his wife.
D Murder of his wife and voluntary manslaughter of the other man.
The correct answer is: Murder of his wife and voluntary manslaughter of the other man.
Discussion of correct answer: The husband killed the other man while in the heat of passion. There was adequate provocation, which will reduce this intentional killing from murder to manslaughter. However, the man did not initially kill his wife while under this provocation. Instead, he went to a bar, had several drinks, and then returned home to kill his wife. A reasonable person would have cooled off in this period. Therefore, the murder of the wife will not be reduced to manslaughter.
A defendant ran an illegal bookmaking operation out of his garage. The police suspected him, but he rarely left his home, and he sent his wife out to make deliveries and pick-ups related to his operation. A parole officer noticed the wife making a delivery of an envelope of cash to a person known to work in illegal bookmaking. The parole officer went to the defendant’s house and demanded to search the house. The defendant refused, saying the officer needed a warrant. The officer showed the defendant his parole documents which included a condition of parole that the defendant agree to consent to unwarranted searches without probable cause. The defendant argued that this did not trump his constitutional rights and that he absolutely refused to provide consent to any search without a warrant. The officer conducted the search anyway, and found evidence related to the bookmaking operation. The defendant argues that the evidence recovered in the search is inadmissible.
Will the court admit the evidence?
A No, because the defendant did not consent to the search.
B No, because the officer lacked reasonable suspicion.
C Yes, because the officer had reasonable suspicion.
D Yes, because the officer had probable cause.
The correct answer is: Yes, because the officer had reasonable suspicion.
Discussion of correct answer: The Supreme Court held in United States v. Knights [534 U.S. 112 (2001)] that, where it is a condition of a defendant’s parole that he consent to unwarranted searches without probable cause, a search is proper when there is reasonable suspicion to conduct a search. Here, the defendant did have such a condition of parole, and the officer had reasonable suspicion to conduct the search based on witnessing the delivery by the wife. As a test tip, a search by a parole officer is almost always valid in an MBE question because the parole officer will only need to have reasonable suspicion, rather than probable cause to conduct a search.
Two custodians at an art museum worked late one night to clean up after the installation of a new exhibit. The first custodian noticed that the installation team had forgotten to install a small photograph. He pointed it out to the second custodian who said that
the photo was actually quite valuable. The first custodian commented that if they both kept quiet about it, then at the end of the shift they could take it with them, sell it on the black market and split the profits. The second custodian laughed and said they probably
could. At the end of their shift, the first custodian took the photo, later sold it on the black market, and the next week handed the second custodian a $100 bill, although the second custodian had no idea what the money was for. Both custodians were charged with conspiracy and larceny. The first custodian was acquitted of conspiracy but convicted of larceny.
What is the most likely outcome of the second custodian’s trial?
(A) He will be found guilty of both larceny and conspiracy.
(B) He will be found guilty of larceny, but not guilty of conspiracy.
(C) He will be found guilty of conspiracy, but not guilty of larceny.
(D) He will be found not guilty of both larceny and conspiracy.
(D) He will be found not guilty of both larceny and conspiracy.
For a year and a half, an architect and his wife had been going through an ugly divorce. Numerous hearings were held to resolve issues of child custody and visitation rights, and mutual restraining orders were entered. To the architect’s great disappointment, in the final divorce decree, the judge ruled that the architect’s wife would receive title to the house in which she and the architect had lived for the 20 years of their marriage. The architect had invested a great deal of time and energy in renovating the house and had cultivated an award-winning garden in the backyard. He decided that if the judge wouldn’t give him justice, he would seek it himself. That night, he went to his former home at a time that he expected no one to be home and sneaked in through a window, planning to burn the house to the ground. However, just as he was about to start pouring gasoline over the furniture and floors, he heard a noise upstairs. Startled, he ran to the garage and jumped into his ex-wife’s car, using the keys she routinely kept under the floor mat to start the engine and drive away. He planned to drive the car only two blocks, to where his own car was parked. In his haste, the architect accidentally left behind the gas can, which was covered with his fingerprints. He is subsequently arrested and charged with burglary, which this jurisdiction defines as at common law.What is the most likely result at trial?
Guilty, because he intended to burn down the house.
Defendant is a student at a local college. While defendant was at class one day, the student’s landlord decided to search the defendant’s apartment for drugs. In the bedroom, the landlord found a baggie of what appeared to be marijuana. The landlord called the police. When the police arrived, the landlord gave them the suspected marijuana. After a field test confirmed the presence of marijuana, the defendant was charged with unlawful possession of marijuana.
At trial, defendant’s attorney moved to suppress the marijuana. The court should
(A) Grant he motion because the police lacked probable cause to seize the marijuana.
(B) Grant the motion because the police lacked a warrant.
(C) Deny the motion because the field test of the marijuana was not a search within the meaning of the 4th Amendment
(D) Deny the motion.
(D) Deny the motion.
One afternoon, a man was driving his car in the downtown section of a city. Already late for a business appointment, he was becoming very upset because of the heavy traffic. Suddenly, he decided to drive his car along the sidewalk to avoid the traffic jam. While he was speeding down the sidewalk, people were hastily scurrying to avoid his car. His car struck a pedestrian, who had tripped trying to get out of his way, killing her.
Which of the following is the best theory regarding the man’s criminal liability?
A Felony murder, because he was committing an assault with a deadly weapon.
B Transferred intent, because one is presumed to intend the natural and probable consequences of his acts.
C Intentional killing, because the man’s conduct was a substantial factor in causing the pedestrian’s death.
D Commission of an act highly dangerous to life, without an intent to kill but with disregard for the consequences.
The correct answer is: Commission of an act highly dangerous to life, without an intent to kill but with disregard for the consequences.
Discussion of correct answer: Extremely reckless conduct, which creates what a reasonable person should recognize as an unjustifiably high degree of risk of death or serious bodily injury to another, constitutes depraved-heart murder when that conduct causes the death of another, even though there was no intent to kill or do serious bodily injury.
A mechanic was told by the owner of the garage he worked in that he needed to burn down a vacant apartment building owned by the owner or he would lose his job. The mechanic, who had five children, could not afford to lose his job, so he agreed. He learned that a superintendent was still living in the basement of the building and told the owner. The owner threatened to kill the superintendent if he did not move out, but the superintendent would not go. The mechanic pleaded with the superintendent to go, but this did not work either. Finally, the mechanic waited until the superintendent had left on an errand, and started a fire on the top floor and then returned to his home. Watching the news with his children later that night, the mechanic saw that the superintendent had died in the fire when he had entered the building to retrieve his stamp collection. The mechanic is charged with felony murder.
Will he be found guilty?
A No, because he believed the superintendent would not in the building during the resulting fire.
B No, because the mechanic had reached a place of temporary safety by returning home.
C Yes, because the mechanic acted with intent to cause serious bodily harm.
D Yes, because the superintendent’s death was a foreseeable result of the arson.
The correct answer is: Yes, because the superintendent’s death was a foreseeable result of the arson.
Discussion of correct answer: Although the mechanic took steps to avoid injuring the superintendent, the mechanic is guilty of felony murder because he committed the felony of arson and this resulted in the death of the superintendent in the building. While the mechanic had intended for the superintendent to be away from the building, it was nonetheless foreseeable that a death would occur as a result of the arson.
A defendant had loaned his landlord some of his favorite record albums one night, and told her she could keep them for a couple months. A couple weeks later, the landlord and the defendant had a fight over unpaid rent, and the landlord told the defendant he was never getting his records back until he paid the rent. The defendant, upset that the landlord had turned his friendly gesture into a bargaining tool, waited until the landlord was away one night, and then broke into the landlord’s house to retrieve his records. He did not realize the landlord’s husband was in the house, and the husband, not recognizing the defendant, attacked him. The defendant fought back, and pushed the husband backwards before fleeing. The husband, who suffered from a brain disorder, hit his head on the ground and died from his injuries.
The defendant will likely be found guilty of which of the following crimes?
A Burglary.
B Involuntary manslaughter.
C Voluntary manslaughter.
D Burglary and murder.
The correct answer is: Burglary and murder.
Discussion of correct answer: The defendant is guilty of burglary because he broke into the apartment intending to commit a felony by stealing back his records. Although they belonged to him, the landlord had legal possession of them. The defendant is also guilty of murder, because the husband died during the commission of the defendant’s felony. Test tip: While it is true that you are not guilty of burglary when you break into someone’s home to reclaim your property, an exception to that rule is when you are taking something back when the other person had legal possession of it. Then you would be committing a theft because you are interfering with the other person’s valid right of possession. In this case, since the tenant loaned the records to the landlord for the specific time period of a couple months, the tenant was not permitted to reclaim his property before then.
After learning he had been laid off from his job at a hair salon, a stylist drove to the house of the salon’s owner, placed a homemade explosive device inside, and detonated it, setting the house on fire. As he left the burning house, the stylist saw a police cruiser heading his way. The stylist jumped in his car, and led the police cruiser on a car chase throughout the city. He was able to lose the cruiser, however, and make it back to his apartment safely. Several hours later, the police came to his apartment and demanded he come out. The landlord of the building stepped outside carrying a cooking pot that the police believed to be a homemade bomb. The police shot the landlord in the hand, intending merely to neutralize him, but the landlord died from excessive blood loss and shock. The stylist is charged with arson and felony murder.
What is the stylist’s best defense to the felony murder charge?
A The stylist’s actions did not meet the intent requirement of the underlying felony.
B The stylist did not engage in an inherently dangerous felony.
C The landlord’s death was not a foreseeable outgrowth of the stylist’s actions.
D By returning to his apartment, he had reached a place of temporary safety.
The correct answer is: By returning to his apartment, he had reached a place of temporary safety.
Discussion of correct answer: A requirement for a felony murder charge is that the resulting death must occur during the commission or perpetration of the felony. The felony is deemed to have terminated when the felon has reached a place of temporary safety. If the killing occurs after this point, the defendant can no longer be found guilty of felony murder. Here, when the stylist reached his apartment, he was in a place of temporary safety, so he cannot be found guilty for the death of the landlord, which occurred hours later.
A senior associate in a law firm was conducting spot checks of other associates as they left the office to determine who had taken some documents that were missing from the firm’s library. A young associate was leaving his office for the day when he was stopped by the defendant, who asked him to open his briefcase.
The senior associate explained that some files from the law office had been stolen. Startled, the young associate refused to permit the senior associate to look in his briefcase. The young associate said that he had personal papers in his briefcase and he didn’t want the senior associate to see them. Having his suspicions aroused, the senior associate raised his clenched fist and exclaimed, “If you don’t hand over that briefcase, I’m going to punch you in the mouth.” The young associate, who, unknown to the senior associate, suffered from a serious heart condition, had a seizure and died as a result of fright produced by the threatened attack. Which of the following is the most serious crime for which the senior associate should be found guilty? (A) Assault. (B) Voluntary manslaughter. (C) Involuntary manslaughter. (D) Murder.
(C) Involuntary manslaughter.
A defendant noticed his neighbor driving around in a vintage hot rod that he had recently purchased, and was determined to get his hands on the hot rod for himself.
In which of the following factual scenarios is the defendant most likely to be found guilty of the crime of false pretenses?
A The defendant offers to trade his truck, which he describes as the “fastest truck in the entire state,” to the neighbor for the hot rod. The neighbor agrees, and they sign over the respective titles to one another, but the neighbor soon learns that there are other trucks in the state that are faster.
B The defendant tells the neighbor that he would like to borrow the car to show it to the children at the school where he volunteers. The neighbor agrees. The defendant finds the title in the hot rod’s glove apartment and signs it over to himself.
C The defendant, a mechanic, offers to examine the hot rod for free. The neighbor agrees. The defendant tells the neighbor that the motor is shot and will be very expensive to replace, which is a lie. The defendant offers to buy the hot rod for less than it is worth, and the neighbor agrees, signing over title in exchange for the defendant’s cash offer.
D The defendant offers to take care of the car while the neighbor is on vacation. The defendant pours sand into the hot rod’s gas tank. When the neighbor returns, he cannot figure out why the hot rod is malfunctioning. He offers to sell it to the defendant at a greatly reduced cost, and the defendant accepts. The neighbor passes title to the defendant.
The correct answer is: The defendant, a mechanic, offers to examine the hot rod for free. The neighbor agrees. The defendant tells the neighbor that the motor is shot and will be very expensive to replace, which is a lie. The defendant offers to buy the hot rod for less than it is worth, and the neighbor agrees, signing over title in exchange for the defendant’s cash offer.
Discussion of correct answer: A defendant is guilty of false pretenses where he makes a false representation of fact causing the victim to pass title to him. Even though the defendant paid money for the car, he is nonetheless guilty of false pretenses as he made a fraudulent statement in order to pay less than the car is worth and receive title.
A professional hitman heard that a gangster wanted to take revenge on a man who had betrayed the gangster’s trust by stealing from him, and then gone into hiding to prepare an escape plan out of the country. The gangster had offered to pay $100,000 to whoever killed the man. The hitman made plans to find the man, and asked a friend for help in locating the man. The friend gave the hitman some information regarding the man’s whereabouts, but then decided he might as well try to kill the man himself and collect the $100,000. The hitman found the man in the public library doing some research on small, desert islands and shot the man in the stomach. Although hurt and bleeding, the man was able to disappear into the book stacks before the hitman could finish the job. The man escaped from the library only to find the friend waiting outside. The friend also shot the man in the stomach, but the man once again escaped the scene before the friend could finish him off. The man made his way to a police station, where he died. Medical examiners determined that the man died from both bullet wounds and resultant bleedings, but neither shooting would have killed him on its own.
The hitman and the friend are put on trial for murder.
How will the court likely resolve the case?
A Both the hitman and the friend will be found guilty.
B Only the hitman will be found guilty.
C Only the friend will be found guilty.
D Neither the hitman nor the friend will be found guilty.
The correct answer is: Both the hitman and the friend will be found guilty.
Discussion of correct answer: When there are multiple causes or other parties responsible for the criminal result, courts will still find a defendant responsible if the defendant’s act was a substantial factor causing the criminal fact. Although neither the hitman or the friend alone would have caused the death of the man, both of them shot the man, and the shootings were a substantial factor in the man’s death, and they will both be found guilty. As a test tip: when a defendant satisfies the mens rea and actus reus elements of specific intent murder at the exact same moment, he will be guilty of that crime. In this case, both the hitman and the friend had the specific intent to kill the man. Both of them shot the man at different times and both of their actions were a substantial factor that caused the man’s death. Therefore, both are guilty of murder.
One afternoon, a man was driving his car in the downtown section of a city. Already late for a business appointment, he was becoming very upset because of the heavy traffic. Suddenly, he decided to drive his car along the sidewalk to avoid the traffic jam. While he was speeding down the sidewalk, people were hastily scurrying to avoid his car. His car struck a pedestrian, who had tripped trying to get out of his way, killing her.
Which of the following is the best theory regarding the man’s liability?
a. Felony murder, because he was committing an assault with a deadly weapon.
b. Transferred intent, because one is presumed to intend the natural and probable consequences of his acts.
c. Intentional killing, because the man’s conduct was a substantial factor in causing the pedestrian’s death.
d. Commission of an act highly dangerous to life, without an intent to kill but with disregard for the consequences.
d. Commission of an act highly dangerous to life, without an intent to kill but with disregard for the consequences.
A defendant committed a string of violent armed robberies at fast food restaurants which resulted in injuries to several victims. Police immediately arrested the defendant for the first armed robbery, but continued to investigate the subsequent incidences. The prosecution did not indict the case until after the police had secured warrants with respect to the subsequent armed robberies. It was determined that the case did not meet the criteria for federal prosecution. It took 40 days from the time of the first arrest to secure all of the applicable warrants and have the case indicted by the grand jury. Due to the judge’s backlog of cases, this case was not scheduled for trial for one year. Once on the trial calendar, the defense attorney filed one motion every month for four months. The defendant remained in custody the entire time, unable to make the high bond. On the day of trial, the defense attorney made a pretrial motion requesting a complete dismissal of all charges and argued that the defendant’s Sixth Amendment right to a speedy trial was violated.
Which of the following should the judge not consider in determining the outcome of the dismissal motion?
A The defendant never asserted his right to a speedy trial.
B The defendant filed a series of pretrial motions and suppression hearings.
C The prejudice to the defendant.
D That the trial did not occur within 70 days of the indictment.
The correct answer is: That the trial did not occur within 70 days of the indictment.
Discussion of correct answer: The right to a speedy trial is a right which is guaranteed by the Sixth Amendment in both the federal and state courts. There are a few distinctions between cases which are heard in the state court and those which are heard in the federal courts. Here, the facts indicate that it was determined that this case was not suited for federal prosecution. Therefore, any rules which apply specifically to the federal courts would not be applicable here. The Speedy Trial Act of 1974 requires that a prosecutor seek a federal indictment within thirty days of the arrest of a defendant and that the trial be held within seventy days of that indictment. If this was a federal case and this type of defense motion was made, the judge would have to determine whether or not the requirements of the Speedy Trial Act were satisfied. However, because this case is not being handled federally, the requirement that the trial begin within seventy days of indictment is irrelevant to the issue of whether the defendant’s right to a speedy trial was violated.
A man has been arrested and charged with homicide for the intentional killing of a neighbor. At trial, the prosecution argued that the man committed murder, and the defendant, in turn, claimed that he should be found guilty of voluntary manslaughter.
Regarding voluntary manslaughter, which of the following statements is most accurate?
A Provocation is only adequate if the criminal defendant was actually provoked and if it would have caused a reasonable person to lose self-control.
B Provocation is adequate if it would cause the defendant to lose self-control.
C As long as the man was still laboring under the heat of passion from the incident, it is irrelevant whether a reasonable person would have cooled off.
D A showing by the man that he was intoxicated would reduce a charge of murder to manslaughter.
The correct answer is: Provocation is only adequate if the criminal defendant was actually provoked and if it would have caused a reasonable person to lose self-control.
Discussion of correct answer: An intentional killing of another done with adequate provocation will constitute a voluntary manslaughter. A criminal defendant who was not actually provoked–and so didn’t act in a heat of passion–would not be able to successfully argue adequate provocation even if a reasonable person would have been provoked in the circumstances. Similarly, a person was actually provoked will not be able to successfully argue adequate provocation unless a reasonable person would have been provoked in the circumstances.
The defendant wanted to steal some documents from his wife’s divorce attorney’s computer, and so he arranged to have a late-night meeting at the attorney’s office. He took with him a small vial filled with a drug used to sedate animals for surgery. The drug was odorless and colorless. When the attorney left the room to take a phone call, the defendant put a small dose of the drug into his drinking cup. After he returned, the attorney took a drink and soon passed out in his chair. The defendant then accessed several documents on the attorney’s computer and left. The defendant was unaware that the attorney had severe allergies which were exacerbated by the drug, causing him to suffer a heart attack and die.
This jurisdiction defines first-degree murder as the intentional killing of another human being with premeditation, and all other murders are classified as second-degree murder. Manslaughter is defined as at common law. This jurisdiction limits felony murder to inherently dangerous felonies and defines those as at common law.
What is the most serious offense of which the defendant can be convicted?
A First-degree murder.
B Second-degree murder.
C Voluntary manslaughter.
D Involuntary manslaughter.
The correct answer is: Second-degree murder.
Discussion of correct answer: Because the defendant did not have any intent to kill the attorney, the issue becomes one of malice. Malice can be express or implied. Implied malice is measured by the circumstances surrounding the defendant’s actions. Here, one could argue that a person lacking sufficient medical training could be malicious in their use of a narcotic. Moreover, while there was no burglary here because this was not a dwelling house, there is a robbery, as the defendant used the narcotic to make the attorney fall unconscious in order to take something from him. The use of fear, force, or intimidation is needed for robbery, and the use of a narcotic would be sufficient to satisfy the force element required. Therefore, the defendant would be guilty of felony murder, which in this jurisdiction would fall within the definition of second-degree murder.
A secretary was asked by her boss to work the coat check at the firm’s annual holiday party. The secretary had been looking forward to the party for months and was hurt and angered that she had to work at the coat check for much of the night. Nevertheless, she agreed. On the night of the party, she checked in the fur coat of the boss’ wife. The secretary had formerly owned a similar fur coat but had to pawn it several years prior. The secretary looked longingly at the fur coat and thought about the fur coat she used to own. The secretary also had several glasses of wine brought to her by her friends from the party. When the secretary was relieved of her coat check duties, she was somewhat intoxicated and picked up the fur coat, believing that it was the fur coat she previously owned and that she still owned it, and wore it home. The next morning, she woke up and saw the fur coat in her closet, and realized it belonged to the wife of her boss. She brought the coat into work and returned it to the boss.
Can the secretary be convicted of larceny?
A Yes, because it was unreasonable for her to believe that the fur coat belonged to her.
B Yes, because voluntary intoxication is not a defense to larceny.
C No, because she believed that the fur coat belonged to her.
D No, because she returned the coat to the boss the next day.
The correct answer is: No, because she believed that the fur coat belonged to her.
Discussion of correct answer: Where a defendant, at the time of the taking, has a good faith belief that he is entitled to possession, there is no intent to permanently deprive, even if that belief is both incorrect and unreasonable. Thus, while it may well have been unreasonable for the secretary to believe that the fur coat belonged to her, she did have such a belief at the time that she took it and thus did not have the requisite intent for larceny.
Which of the following is most likely to be a strict liability offense?
A A city ordinance making it an infraction, for which a $100 fine is payable, to trespass upon the property of another that is posted as private property.
B A state statute providing that the failure to have a current valid inspection sticker on a vehicle is a misdemeanor.
C A state statute providing that it is a felony punishable by two, three, or four years in prison to carry a concealed knife whose blade is longer than three and one-half inches.
D A federal statute which provides that any person who files an income tax return containing false information has committed a felony punishable by imprisonment for five years and up to $5,000 in fines.
The correct answer is: A state statute providing that the failure to have a current valid inspection sticker on a vehicle is a misdemeanor.
Discussion of correct answer: Four categories of crime are likely to be strict liability offenses: regulatory offenses, public welfare offenses, morality crimes, and selling liquor to a minor. A requirement that a vehicle have a valid inspection sticker is a public welfare statute. Two major factors which indicate that a defined crime is a public welfare offense (and, therefore, treated as imposing strict liability) are: (1) that the penalty is not severe; and (2) that the harm to the public from the prohibited behavior is serious. The crime described in this choice–operating a vehicle on public roads without a valid inspection sticker–is only a misdemeanor, and the harm to the public that could result from violation is very serious. It is most likely that a violator would be subject to strict liability–that is, liable without fault or a showing of bad intent.
A restaurant had ceased to be profitable when a competitor opened a buffet restaurant across the street. The restaurateur projected that he would go bankrupt in three months unless his patronage immediately rose to the levels he had enjoyed prior to the opening of the competitor’s restaurant. The restaurateur asked his bookie whether he knew anyone who would do a job, no questions asked, for cash. The bookie said to contact a certain woman who frequented a local bar. The restaurateur went to the bar, met the woman, and offered the woman $5,000 to burn down the competitor’s restaurant. The woman, an undercover police officer, immediately arrested the restaurateur. The restaurateur was subsequently charged with solicitation to commit arson. The jurisdiction has not altered the common law by any statute.Should the restaurateur be found guilty or not guilty?
Not guilty, because the woman could not commit arson by performing the requested act.
The police suspected that a teenager, a 16-year-old high school student, had committed a series of burglaries. Two officers went to the teenager’s high school and asked the principal to call him out of class and to then search his backpack. While the officers waited, the principal took the teenager into the hall, where she asked to look in his backpack. When the teenager refused, the principal grabbed it from him, injuring the teenager’s shoulder in the process. In the backpack, she found jewelry, which she turned over to the officers. The teenager was subsequently charged with committing the burglaries, and now moves to suppress the use of the jewelry as evidence at trial.
How should the court rule?
A Deny the motion, because the search was incident to a lawful arrest.
B Deny the motion, because school searches are reasonable if conducted by school personnel, on school grounds, on the basis of reasonable suspicion.
C Grant the motion, because the search was conducted with excessive force.
D Grant the motion, because the search was conducted without probable cause or a warrant.
The correct answer is: Grant the motion, because the search was conducted without probable cause or a warrant.
Discussion of correct answer: The Fourth Amendment applies only to government, not private, conduct. However, when a private party acts at the direction of a government agent, any search conducted and evidence seized will be subject to Fourth Amendment scrutiny. The police asked the principal to call the teenager out of class and to then search his backpack. There was no warrant or probable cause for this action. Therefore, the motion to suppress the jewelry as evidence should be granted.
A U.S. Marshal tracked down a man suspected of murdering several people. The marshal approached the man in the parking lot of a motel and when the suspect saw the marshal, the suspect started running down an ally. The marshal chased the suspect on foot and the suspect ran onto someone’s porch, broke down the door, and entered the living room. The marshal saw the suspect just as he ran into the house and the marshal ran through the house a few seconds later. He finally caught the suspect in a back bedroom and arrested him. As the marshal walked the suspect out of the house, the marshal saw several scales, small bags, and piles of marijuana on the kitchen table. A woman who was hiding in the kitchen pantry was arrested and charged for possession and intent to sale.
If the woman files a motion to suppress the scales, bags and marijuana, how will the judge rule?
(A) Motion denied, because the house was searched after a lawful arrest was made inside.
(B) Motion denied, because the marshal was in hot pursuit of the suspect and the scales, bags, and marijuana were on the kitchen table.
(C) Motion granted, because of the plain view doctrine.
(D) Motion granted, because the suspect did not have the woman’s consent to enter the home.
(B) Motion denied, because the marshal was in hot pursuit of the suspect and the scales, bags, and marijuana were on the kitchen table.